You are on page 1of 48

Newton’s Laws of Motion 4.

4
Newton’s Laws of Motion
1. FORCE
We know by experience that all bodies in nature interact
in some way with one another. Force is a measure of the
interaction between the bodies. The force may either
produce deformation (change in the size or shape of
When the card holding the coin is flicked , the
bodies) or acceleration (change in magnitude or direction
card flies off and the coin falls in the glass,
of their velocity).
whereas we would expect it to be carried away
It can be described as a push or a pull. The pull is with the card.
always oriented away from the body; and the push is (b) Passengers travelling by train fall forward when
always oriented towards the body. the train applies sudden brake.
F F
Passengers have the same speed as the train.
When the train slows down, due to inertia the
A pulling force A pushing force
upper body of passengers continues to move
with same speed.
Force is a vector quantity. Every force has a definite
direction and the result of its action depends on the Illustration 1
direction and the magnitude of the force. If several forces A well-known lecture demonstration consists of
are applied to a particle, they can be replaced by the hanging a heavy ball from a fixed bar by a thin
resultant force. string. An identical string is connected to a hook on
Unit of force: the bottom of the mass.
kg ⋅ m
newton and (MKS System)
s2

g ⋅ cm
dyne and (CGS System)
s2

1 newton = 105 dyne

2. LAWS OF MOTION
2.1 NEWTON’S FIRST LAW
Concept of inertia Comment on what would happen (a) If the lower
string is pulled slowly (b) if the lower string is
Inertia is the resistance of any physical object to any
suddenly jerked.
change in its state of motion. This includes changes to the
Solution:
object’s speed, direction, or state of rest.
(a) If the lower string is pulled slowly, the upper string
A few examples of concept of inertia could be: eventually breaks, because it supports both the
(a) Coin and glass experiment applied force and the hanging weight.
4.2 Physics
(b) However, if the lower string is suddenly jerked, 2.3 NEWTON’S THIRD LAW
it is the one which breaks. The explanation is If an object exerts a force F on a second object, then the
second object exerts an equal but oppositely directed force
that the inertia of the mass keeps it from moving
–F on the first. This pair of forces are referred to as action
and hence preventing it from stretching the upper and reaction.
string.
Fearth
F man
.These examples strengthen the idea that Object possess a Fsun
tendency to maintain their state of motion and no force is S
F ground
required to maintain the motion.
(a) (b)
Newton stated these ideas as follows:
Forces exists in pairs
When there is no net force on an object, an object at (a) The force exerted by earth on the sun is equal
rest remains at rest, and an object in motion continues and opposite to the force exerted by the sun on
to move along a straight line with constant velocity. the earth. Fearth = –Fsun.
This is also known as the law of inertia. (b) The force exerted by the man on the ground is
For example, we could imagine a spaceship moving equal and opposite to the force acting on the
in space, with its engines shut off and far away from man by the ground. Fman = Fground.
gravitational effects of any other object will keep moving
with constant velocity in a straight line. The spaceship
maintains its motion due to its inertia. No force is required  Newton’s third law action and reaction forces act
to maintain its motion. on different objects. Two forces acting on the same
object, even if they are equal in magnitude and
2.2 NEWTON’S SECOND LAW opposite in direction, cannot be an action-reaction
We know from the first law, what happens when there pair.
is no unbalanced force on an object: its velocity remains
constant. Now let us see What happens when there is an Illustration 2
unbalanced force on an object? Which of the following is the reaction force to the
The Newton’s second Law gives the answer to this gravitational force acting on your body as you sit in
your desk chair?
question, that is, the net force acting on a body will
produce an acceleration. (a) The normal force exerted by the chair
(b) The force you exert downward on the seat of the
The magnitude of the acceleration produced depends
on the quantity of matter being acted upon. The quantity chair
of matter is referred to as the inertial mass. (c) Neither of these forces.
Newton’s second law states the relation between the net Solution:
force and the inertial mass. Action reaction pairs act between a pair of interacting
  objects. In the case for gravitational force the interaction
Σ F = m a is between earth and you, if earth exerts a gravitational
Note that the direction of acceleration is in the direction force on you, the reaction of this force should be the
of the net force. attractive force exerted by you on earth. This force will
In terms of components act on earth.
SFx= max  SFy= may  SFz= maz
Illustration 3
A horse refuses to pull a cart. The horse reasons,
“according to Newton’s third law, whatever force I
 The direction of motion of a particle does not in
exert on the cart, the cart will exert an equal and
general coincide with the direction of the force opposite force on me so the resultant force will be
acting on it. It is the acceleration or rate of change of zero and I will have no chance of accelerating the
velocity, which is related to the force. cart.” What is wrong with this reasoning?
Newton’s Laws of Motion 4.3
Solution: G is universal gravitational constant.
GRAVITATIONAL FORCE DUE TO EARTH
F Consider on object of mass m kept near the surface of
earth. (distance between the center of earth and object can
Ground pushes be approximately taken as the radius of earth ‘R’)
horse forward
f f Earth attracts the object with a force F given by
f GMm
Horse pushes ground F =
R2
The figure is a sketch of a horse pulling the cart. M → Mass of earth
Since we are interested in the motion of the cart, we R → Radius of earth
have circled it and indicated the forces acting on it.
The force exerted by the horse is labelled F and the GM
We note that is a constant m
horizontal force exerted by the ground labeled f (for R2
friction). The cart will accelerate if F is greater than f. which is known as acceleration due F = mg

The forces on the horse in horizontal direction are to gravity denoted by the symbol ‘g’.
reaction force F and the force f ′ which the ground g is approximately equal to F = mg
exerts as a reaction to the horse pushing the ground 9.81 m/s2 near the surface of earth. M
in backwards direction. The horse will move if f ′ is
Force with which the earth attracts R
greater than F.
the object is equal to F =
Horse’s argument is wrong because the two forces of
 GM 
magnitude F act of separate objects and hence they m  2  = mg . It is also known as
 R 
can’t cancel each other.
To move the horse has to push the ground with a force the weight of the object.
f ′ larger than F and then the system will move forward. According to Newton’s third law, the object also exerts
an attractive force on earth equal to its weight as shown
3. TYPES OF FORCES in the diagram.
Non contact forces
3.2 CONTACT FORCE
When two objects come in contact, they exert forces
Gravitation Electromagnetic Nuclear on each other which arise due to short range atomic or
Contact forces molecular interactions. These are electro magnetic in
nature.
A book resting on a table is supported by very large
Normal Tension Friction Spring number of electromagnetic interactions between the atoms
Gravitational force is the weakest of the other forces in the adjacent surface layers of the book and the table.
like electro-magnetic or nuclear To simplify the analysis, the resultant of all these forces
All contact forces are electromagnetic in nature. can be called a contact force.
3.1 GRAVITATIONAL FORCE The component of contact force along the normal to
the contact surface is called Normal force.
Newtons law of gravitation
The component of Contact force along (tangential) the
Two particles of mass m1 and m2 are separated by a contact surfaces is called friction.
distance r
Contact force
F F
Normal
m1 m2 force
r
Friction
Each particle exists an attractive force of magnitude F on
the other which is given by the result
Gm1m2 3.2.1 Normal Force
F =
r2 It is the Component of contact force along the normal to
G = 6.67 × 10–11 m2/kgs2 the contact surface.
4.4 Physics
Following examples show how normal force is marked Example 5:
in free body diagrams. Block pushes the table down with Normal forces pass
through the center
a force N, table pushes the block up with force of same
magnitude. N2 N1

Example 1:
m Object 1 2 2 2

N N1 N2

m N 3.2.2 TENSION
Table
Ground Observe the situation where a block is pulled with a force
N1 N2 F using a string or rope.
N1 N2
m F
Ground
Similarly Normal forces are drawn between the table N Force exerted
legs and ground. by
string F1
F1 F
Example 2:
A man pushing blocks mg
When we apply the pulling force F, the string stretches
m1 m2 and at any smaller section as shown in diagram, we have
Ground
internal pulling (tensile) forces. These forces are called
Tension.
N1 N2 N2 F1 F1 F1 F2 F2 F
N1
N3
(i) if string is massless then internal forces or
N4
N5 tension throughout the string is same.
N3 N4 N5 (ii) if string has mass, tension at different sections
may be different. We apply newton’s laws to
Ground
calculate tension at any point.
Example 3: (iii) if we have a rod instead of string we could have
A block kept on an inclined plane compressive as well as tension forces.
Drawing forces with systems involving pulleys
m
Pulleys could be used to change the direction of force.
N

m
N

F
Clearly the normal force is
perpendicular to contract surface.

Example 4:
In particle dynamics we assume the pulley to be smooth
and therefore string/rope slips on it without any friction.
N2
2 Tension in the string doesn’t change as it passes around
a pulley
N1
3.4 SPRING FORCE
1 Whennoforce actsona spring,it is T1

If the contact is at a point. The 2 in relaxed condition i.e., neither


T1 = T2
normal force is perpendicular to N2 compressed nor elongtaed.
N1
the extended surface at point of Consider a spring attached to
contact. 1
fixed support at one of its end T2
Newton’s Laws of Motion 4.5
and the other end is free. It we neglect gravity, it remains elongated and displacement x of its free end is called
in relaxed state. When it is pushed by a force F, it is elongation. Various forces developed in these situations
compressed and displacement x of its free end is called are shown in the figure.
compression. When the spring is pulled by a force F, it is
Relaxed spring x = 0 Equilibrium or
mean position

Compression
Compressed spring x
F
Force by spring
on the wall
F F F
Force by Force by Force by
wall on hand spring on
spring on spring hand Elongation
x

Elongated spring
F
Force by wall
on spring F F F
Force by spring Force by spring Force by hand
on wall on hand on spring
The force applied by the spring on the wall and the force applied by the wall on the spring make a third law
action‑reaction pair. Similarly, force by hand on the spring and the force by spring on the hand make another third law
action reaction pair.
HOOKE’S LAW
How spring force varies with deformation in length x of the spring is also shown in the following figure.
Relaxed spring x = 0 Equilibrium or
mean position

Compression
Compressed spring x

Elongation
F
x

Elongated spring

Spring force

F
Compressions Extension
–x x

–F

The force F varies linearly with x and acts in a direction SI unit of spring constant is newton per meter or (N/m)
opposite to x. Therefore, it is expressed by the following
equation

F = − kx  If a spring is stretched/compressed by an amount ‘x’ a
force of magnitude ‘kx’ acts the two ends of the string.
Here, the minus (–) sign represents the fact that force F Relaxed
is always opposite to x.
Kx Kx
The constant of proportionally k is known as force Compression by ‘x’
constant of the spring or simply as spring constant. The Kx Kx
Elongation by ‘x’
slope modulus of the graph equals to the spring constant.
4.6 Physics
INTEXT EXERCISE: 1
1. You are standing in a moving bus, facing forward, 5. Two blocks, A and B, are being pulled to the right
and you suddenly fall forward as the bus comes to along a horizontal surface by a horizontal 100–N
an immediate stop. The force acting on you that pull, as shown in the figure. Both of them are moving
causes you to fall forward is together at a constant velocity of 2.0 m/s to the right,
(a) the force of gravity. and both weigh the same.
(b) the normal force due to your contact with the
floor of the bus. A

(c) the force due to static friction between you and B 100 N pull
the floor of the bus.
(d) No forces were acting on you to cause you to
fall.
Which of the figures below shows a correct free-
2. An object is moving forward with a constant body diagram of the horizontal forces acting on the
velocity. Which statement about this object MUST upper block, A?
be true?
(a) A 100 N
(a) The net force on the object is zero.
(b) The net force on the object is in the forward (b) 100 N A 100 N
direction.
(c) No forces are acting on the object. (c) A
100 N
100 N
(d) The acceleration of the object is in the forward (d) A (No horizontal forces act on A.)
direction.
6. A satellite is in orbit around the earth. Which one
3. A ball is tossed vertically upward. When it reaches
feels the greater force?
its highest point (before falling back downward)
(a) the velocity is zero, the acceleration is directed (a) the satellite because the earth is so much more
downward, and the force of gravity acting on the massive
ball is directed downward. (b) the earth because the satellite has so little mass
(b) the velocity is zero, the acceleration is zero, and (c) Earth and the satellite feel exactly the same
the force of gravity acting on the ball is zero. force.
(c) the velocity is zero, the acceleration is zero, and (d) It depends on the distance of the satellite from
the force of gravity acting on the ball is directed
Earth.
downward.
(d) the velocity and acceleration reverse direction, 7. You are in a train traveling on a horizontal track and
but the force of gravity on the ball remains notice that a piece of luggage starts to slide directly
downward. toward the front of the train. From this observation,
4 A dog is standing in the bed of a pickup truck. The you can conclude that this train is
bed is coated with ice, causing the force of friction (a) speeding up.
between the dog and the truck to be zero. The truck (b) slowing down.
is initially at rest, and then accelerates to the right,
(c) changing direction.
moving along a flat road. As seen from a stationary
observer (watching the truck move to the right), the (d) speeding up and changing direction.
dog
8. You push on box G that is
(a) does not move left or right, but the back of the
next to box H, causing Push
truck moves towards the dog. G H
both boxes to slide along
(b) moves to the right, but not as quickly as the
the floor, as shown in the
truck is moving to the right, causing it to slide
towards the back of the truck. figure. The reaction force to your push is
(c) moves to the right at the same rate as the truck, (a) the push of box G on box H.
so it doesn’t slide. (b) the push of box H on box G.
(d) moves to the left, as the truck moves to the right, (c) the push of box G against you.
causing the dog to slide towards the back of the
(d) the upward force of the floor on box G.
truck.
Newton’s Laws of Motion 4.7
9. A book is lying on an inclined plane having the fireman. When the force on his hands equals his
inclination to the horizontal θ°. What is the angle weight, what happens to the fireman?
between the weight of the book and the normal (a) The fireman comes to a stop.
reaction of the plane on the book? (b) The fireman descends with slower and slower
(a) 0° (b) θ° speed.
(c) (180 – θ°) (d) 180° (c) The fireman descends with a smaller but non-
10. A fireman is sliding down a fire pole. As he speeds zero acceleration.
up, he tightens his grip on the pole, thus increasing (d) The fireman continues to descend, but with
the vertical frictional force that the pole exerts on constant speed.

Answer Key
1. (d) 2. (a) 3. (a) 4. (a) 5. (d) 6. (c) 7. (b) 8. (c) 9. (c) 10. (d)

4. EQUILIBRIUM OF A PARTICLES Solution:


A particle body is said to be in equilibrium if the net The free body diagram of the joint O is drawn as shown
force acting on the body is zero. That is, in the figure

∑F x =0 T2 cos 30°
T2

∑F y =0 y 30°

∑F z =0
x
To analyze the equilibrium of a body, the following T1 O T2 sin 30°
steps are adopted
(i) Mentally isolate the body from its environment
or surrounding.
100 N
(ii) Identify the forces acting on the body and
draw a diagram showing every force acting on Applying equations for equilibrium.
the body. This diagram is known as free body
SFx = 0 T2 sin 30° − T1 = 0 ...(i)
diagram (F. B. D.)
(iii) Select two mutually perpendicular axes, usually SFy = 0 T2 cos 30° – 100 = 0 ...(ii)
called x and y axes and resolve the components
of forces along these axes. 100 200
Thus T2 = = N
cos 30° 3
(iv) Set the algebraic sum of all x-components
and y-components of forces equals to zero
Substituting the value of T2 in equation (i), we get
i.e. SFx = 0 and SFy = 0
Illustration 4 100
T1 = T2 sin 30° = N
A block of mass 10 kg is suspended with two strings, 3
as shown in the figure. Find the tension in each
string. (g = 10 m/s2) Illustration 5
In the situation shown in figure the
T2 block of weight W is suspended from
°
30

a string. The pulley is frictionless


T1 O and massless. Calculate the tension
in the string and the force exerted W
10 kg by the clamp on the pulley.
4.8 Physics
Solution: T Solution:

F.B.D. of Block F.B.D. of 10 kg block


N1
SFy = 0  ⇒  T – W = 0
W 100 N
\ T = W T N2

F.B.D. Pulley
10 g
SFx = 0  ⇒  Nx= T Nx
N1 = 10 g = 100 …(1)
SFy = 0  ⇒  Ny= T Nv T
N2 = 100 N …(2)
The net reaction force N = N x2 + N y2 =
T 2
F.B.D of 20 kg block
N2 = 50 sin 30° + N3
or N = W 2
\ N3 = 100 – 25 = 75 N …(3)
N4
Illustration 6 50 N

In the diagram shown, the blocks A and B are 30°


connected by light string and are at rest. If WA = 10 N
then find the tension in the string and WB . N2 N3

A
20 g
B
30° and N4 = 50 cos 30° + 20 g
N4 = 243.30 N
Solution:
N
Illustration 8
T
Find the magnitude of the horizontal force F
T
required to keep the block of mass m stationary on
10 sin 30° the smooth inclined plane as shown in the figure.
30° 10 cos 30° = 53
WB
10 N T – WB = 0 F
m
F.B.D. of Block A    F.B.D. of Block B
Since the system is at rest,
π 
T = 10 sin
=5N
6
and WB = T = 5 N Solution:
The forces acting on the block are shown in the free
Illustration 7 body diagram.
N F cos 
Two blocks are kept in contact as shown in figure. N
Find F
x 90 – 
(a) forces exerted by surfaces (floor and wall) on y

blocks. F sin 
(b) contact force between two blocks. mg
mg sin  mg cos 
50 N
SFx= 0
30°
F cos q − mg sin q = 0
100 N mg sin θ
10 kg 20 kg or F = = mg tan q
cos θ
Newton’s Laws of Motion 4.9
Illustration 9 Solution:
A block of mass M is connected to a pulley and a T1 T2 T2

light spring as shown in figure. The stiffness constant


of the spring is k. Find the extension in the spring
when the whole system is in equilibrium. The pulley Mg T1
is smooth and massless. T1 = Mg 2T2 = T1
F.B.D. of Block F.B.D. of Pulley

k Let x be the extension in the spring, then


T Mg
or T2 = 1 =
2 2
T Mg
kx = T2  or  x = 2 =
M k 2k

INTEXT EXERCISE: 2

1. Two objects, each of woman exerts on the crate, C is the vertical contact
weight W, hang force exerted on the crate by the floor, and W is
vertically by spring the weight of the crate. How are the magnitudes
scales as shown in the W W of these forces related while the woman is trying
figure. The pulleys and unsuccessfully to lift the crate?
the strings attached to the objects have negligible (a) P + C = W (b) P + C < W
weight, and there is no appreciable friction in the
(c) P + C > W (d) P = C
pulleys. The reading in each scale is
(a) W (b) less than W 5. Two weights are connected by a massless wire and
pulled upward with a constant speed of 1.50 m/s by
(c) 2W (d) more than 2W
a vertical pull P. The tension in the wire is T (see
2. A fish weighing 16 N is weighed using two figure). Which one of the following relationships
spring scales, each of negligible weight, as between T and P must be true?
shown in the figure. What will be the readings Pull P
of the scales?
(a) The bottom scale will read 16 N, and the
25 N
top scale will read zero.
(b) Each scale will read 16 N. Tension T
(c) The scales will have different readings,
but the sum of the two readings will be 16 N.
100 N
(d) Each scale will read 8 N.

3. A traffic light weighing 100 N is supported by two


ropes as shown in the figure. The tensions in the (a) T > P (b) T = P
ropes are closest to (c) P + T = 125 N (d) P = T + 25 N
37° 37°
6. A mass M of 100 kg is R

suspended with the use of 45°


90° C
strings A, B and C as shown
B
in figure, where W is the W A
vertical wall and R is a rigid
(a) 50 N (b) 56 N horizontal rod. The tension M = 100 kg

(c) 63 N (d) 83 N in the string B.


(a) 100 gN (b) zero
4. A woman is straining to lift a large crate, without
100
success because it is too heavy. We denote the forces (c) 100 2 gN (d) gN
on the crate as follows: P is the upward force the 2
4.10 Physics
7. An iron sphere weighs 10 N A B
9. Two bodies of mass 4 and 6 kg are attached to the
and rests in a V shaped trough ends of a string passing over a pulley. The 4 kg mass
whose sides form a angle is attached to the table top by another string. The
60°. What are the normal
60° tension in this string T1 is equal to
forces exerted by the walls
on the sphere in case as
shown in figure.
(a) RA = 10 N, RB = 0 (b) RA= RB = 10 N T
T
10 4 kg
(c) RA = 10√3, RB =
3 T1 6 kg

(d) RA = RB = 5 N
8. In the system shown in the adjoining (a) 19.6 N (b) 25 N
T3
figure, the tension T2 is (c) 10.6 N (d) 10 N
1 kg
(a) g T2 10. In the above question, if string between 4 kg and
(b) 2 g 2 kg surface is cut, findout acceleration of system
(c) 5 g T1 (a) 1.5 m/s 2 (b) 1.96 m/s 2
(d) 6 g 3 kg
(c) 1 m/s 2 (d) 5 m/s 2

Answer Key
1. (a) 2. (b) 3. (d) 4. (a) 5. (d) 6. (a) 7. (b) 8. (c) 9. (a) 10. (b)

5. DYNAMICS OF PARTICLES 1. Identify the block as the y

Writing Equations of Motion system as we have to


1. Identify the system first then choose the calculate its acceleration.
reference frame depending upon the motion of 2. Choose x and y axes along
the system. and perpendicular to the
x
2. Draw the F.B.D. of the system and resolve all incline plane.
the forces along the chosen axes.
Note: Since we are aware the direction the block will
3. Then add all the forces along each perpendicular
move, we choose one of the directions to coincide
axes taking care of the respective sign and equate
to mass times the acceleration of the system in with the direction of motion. In general we
that particular direction. can choose any set of N
Let us consider an example perpendicular directions.
A suitable choice will
however help us solve the
m
problem faster.
3. Draw F.B.D. of the blocks mg
 m. Here only the weight
mg and the normal reaction N are acting on the
A block of mass m is placed over a fixed smooth block.
inclined plane having inclination q. We have to find the 4. Resolve forces along the x and y-directions.
acceleration of the block.
Newton’s Laws of Motion 4.11
N
On adding equation (i) and (ii), we get
10 = (3 + 2) a

mg sin  
or a = 2 m/s2
mg mg cos  (b) Substituting the value of a in equation (ii), we get
5. Write equation of motion in x and y directions N = 2(2) = 4 newton
separately taking care of the respective signs of Note that one can easily find the acceleration of the
forces blocks by considering both blocks together as shown
    mg sin q = max (accelerated) along the x-axis in the figure.

N – mg cos q = may = 0 (equilibrium) along the y-axis


10 N N
\ N = mg cos q 3 kg N 2 kg
mg sin θ
and ax = = g sin θ
m The normal reaction which exists in equal
and opposite pairs get cancelled when we
consider the whole system.
Illustration 10 In this situation, applying Newton’s Second law along
Two blocks of mass 2 kg and 3 kg are placed side by the x-axis, we get
side on a smooth surface as shown in the figure. A SFx = max
horizontal force of 10 N is applied on the 3 kg block
or 10 = (3 + 2) a  ⇒  a = 2 m/s2
10 N
3 kg
2 kg To find out the normal reaction, one can imagine the
situation like this: when a force of 10 N acts on the
(a) Find the acceleration of each block. system of blocks each block accelerates with 2 m/s2.
(b) Find the normal reaction between two blocks. In order to accelerate the 2 kg block with 2 m/s2 we
require a force of 4 N. Since the only force accelerating
Solution:
the 2 kg block is the normal reaction force. Normal
N1 reaction force must be 4 N.

y
10 a1x = a1 Illustration 11
x
N In the arrangement shown in figure. The strings
a1y = 0 are light and inextensible. The surface over which
3g
blocks are placed is smooth. Find.
Free Body Diagram of 3 kg mass (a) the acceleration of each block
N2 (b) the tension in each string.

4 kg F = 14 N
a2x = a2 2 kg 1 kg
N

a2y = 0 Solution:
2g
(a) Let ‘a’ be the acceleration of each block and T1 and
Free Body Diagram of 2 kg mass T2 be the tensions, in the two strings as shown in
Since the blocks cannot move normal to the surface, figure.
therefore. a1y = 0 and a2y = 0. y

Using SFx = max for each block, we get


T2 T1
For 3 kg block 10 − N = 3a1...(i) 4 kg F = 14 N
2 kg 1 kg
For 2 kg block N = 2a2...(ii) x

(a) Since the blocks are moving together, therefore, a1 aking the three blocks and the two strings as the
T
= a2 = a system.
4.12 Physics
a
T = 8 N
Assume the tension at the midpoint of the string is T1.
4 kg 2 kg 1 kg
F = 14 N We draw the free body diagram of the right half of the
string as given below

∑F
a
Using x = max T1 0.25 kg
or 14 = (4 + 2 + 1) a T1
9N

14
or a= = 2 m/s2. Using the Newton’s second law
7
9 – T1 = (0.25)(a)
(b) Free body diagram (showing the force in x-direction
only) of 4 kg block and 1 kg block are shown in Mass of the right half of string is 0.25 kg and its
figure. acceleration is same as the whole system
y Solving we get
a = 2 m/s
2
a = 2 m/s2
T1 = 8.5 N
T2 T1 F = 14 N
4 kg 1 kg
x Illustration 13
Using ∑F x = max Two blocks of mass m1 and m2 are attached at the
ends of an inextensible string, which passes over a
For 1 kg block, F – T1 = (1) (a)
smooth massless pulley. If m1 > m2, find
or 14 – T1 = (1) (2) = 2
∴ T1 = 14 – 2 = 12 N
For 4 kg block, T2 = (4)(a)
∴ T2 = (4)(2) = 8 N.

Illustration 12 m2

A block of mass 4 kg is pulled by a heavy uniform


m1
rope of mass 0.5 kg with a horizontal force of 9 N as
shown in figure. Find the force exerted on the block (a) the acceleration of each block
by the rope. Find the tension at the midpoint of the
string. The surface is smooth. (b) the tension in the string.
Solution:
M 9N
The free body diagram of each block is shown in the
figure.
Solution: T T
Let the force exerted by the rope on the block be T and
let the common acceleration of the block and rope be a.
N a2 m2 m1 a1
a
a
T T 9N
F.B.D of rope m2g m1g
F.B.D. of m2 F.B.D. of m1
40 N
F.B.D. of block Note about assumptions
Applying Newton’s Second Law The block m1 is assumed to be moving downward and
the block m2 is assumed to be moving upward. It is
on the block T = 4a ...(i) merely an assumption and it does not imply the real
direction of motion. If the values of a1 and a2 come out
on the rope 9 – T = (0.5)a...(ii)
to be positive then the assumed directions are correct;
Solving equations (i) and (ii) for the value of T, we get otherwise the bodies move in the opposite directions.
Newton’s Laws of Motion 4.13
Since the two blocks are connected by non-extendable Applying Newton’s Second Law
string, the magnitudes of their accelerations must be Mg – N = Mao
equal Let us assume a1 = a2 = a. or N = M(g – ao)
Applying Newton’s second Law on Thus, when the elevator accelerates down, the
Block m1 m1g − T = m1a1...(i) apparent weight of the man is lesser than his true
weight by Ma0.
Block m2 −m2g + T = m2a2...(ii)
N
Substituting a1 = a2 = a in equations (i) and (ii) and
after solving them, we get

 m − m2 
a =  1 g
ao
 m1 + m2 
2m1m2
also T = g.
m1 + m2 Mg

F.B.D. of man w.r.t. ground


Apparent weight in an Accelerating Elevator
Illustration 14 If a0 = g, as it would if the elevator were in free fall,
the man would be apparently weightless.
A man of mass M stands on a scale in N
If elevator is moving with uniform motion, then
an elevator, as shown in the figure.
a0= 0 and therefore the reading in the weighing
Find the scale reading when the
machine will be equal to the true weight.
elevator is accelerating with a0.
(i) upward and ao Illustration 15
(ii) downward?
What will be the reading of spring balance in the figure
Solution: shown in following situations. (g = 10 m/s2)
Mg
The reading of the weighing scale is equal to the normal
force exerted by the person on the weighing scale.
a v
(a) Since the man is stationary relative to the elevator,
he is also accelerating up with a0. The forces on the
M = 10 kg
man are reaction N (exerted by the scale platform
on which it rests) and Mg, the force of gravity. (i) a = 0, v = 0
N (ii) a = 0, v = 2 m/s
(iii) a = 0, v = –2 m/s
(iv) a = 2 m/s2, v = 0
ao (v) a = –2 m/s2, v = 0
(vi) a = 2 m/s2, v = 2 m/s
(vii) a = 2 m/s2, v = –2 m/s
Mg (vii) a = –2 m/s2, v = –2 m/s
F.B.D. of man w.r.t. ground Solution:
Applying Newton’s Second Law
N – Mg = Mao Reading of spring balance is equal to the tension
or N = M(g + ao) (force acting at the both ends) in spring balance which
The force exerted by the man on the scale is equal doesn’t depend on velocity of elevator but depends on
and opposite to N and determines the reading on acceleration.
the scale or the apparent weight. Thus, when the We draw free body diagram of the spring balance and
elevator accelerates up, the apparent weight of the mass system.
man is greater than its true weight by Ma0.
For cases (i), (ii) and (iii), the acceleration is zero
(b) When the elevator accelerates downward with a0
the free body diagrams can be drawn as shown in For the block,
the figure. T – 100 N = 0
4.14 Physics
T = 100 N Solution:
T
N T

T
a T N

100
300 N 600 N

For cases (iv), (vi) and (vii) the elevator is accelerating (b) For correct reading N = 600 Newtons, now man will
upwards with acceleration of 2 m/s2 pull the rope with a larger force so that the whole
For the block, system will accelerate up with an acceleration ‘a’
T – 100 N = 10(2) Using newton’s second law
T = 120 N T − ( N + 300) =30 a 
For cases (v) and (viii), the elevator is accelerating
 N = 600
T + N − 600 = 60 a 
downwards with acceleration of the 2 m/s2. Magnitude
and direction of velocity are not important. solving T = 1800 N

For the block,


Problems related to cutting strings and/or springs
100 N – T = 10(2)
If any change is made in a the system (cutting any
T = 80 N string or spring) the tension forces is the strings change
instantaneously whereas the spring forces of the attached
Illustration 16 springs doesn’t change.
Figure shows a man of mass 60 kg standing on a Illustration 17
light weighing machine kept in a box of mass 30 kg. Find the acceleration of each block just after cutting
The box is hanging from a pulley fixed to the ceiling
through a light rope, the other end of which is held
by the man himself. If he manages to keep the box
at rest 1

(a) the string connecting 1 kg block to ground


(b) the spring attached to 2 kg block
Solution:

Free body diagram before cutting


Fs = 20 N
T = Fs
(a) what is the weight shown, by the machine? Also T = T1 + 10
(b) what force should he exert on the rope to get his Fs Fs
correct weight on the machine? T

Solution:
1 2

(a) Weight shown by machine is equal to normal force N.


10 N
Due to equilibrium, T1 Fs
20 N

N + T = 600 This means T1 = 10 N


T = N + 300 When string is cut T1 = 0 but the Fs doesn’t change
solving, N = 150 Newtons instantaneously
Newton’s Laws of Motion 4.15
T = Fs = 20 N Fs = 20 N When spring is cut Fs = 0
T = Fs = 0 Fs = 0
1 2 a=0
1 a=g 2 a=g
10 N
20 N
10 N
T1
20 N
acceleration of 1 kg block:
T1 will become zero and both 1 kg and 2 kg blocks
20 − 10
a = = 10 m/s2 will start falling down with acceleration ‘g’
1

INTEXT EXERCISE: 3
1. A particle of mass 50 g moves on a straight line. g g
The variation of speed with time is shown in the (a) (b)
10 20
figure. The force acting on the particle at t = 2, 4 and
6 seconds is g g
(c) (d)
v(m/s) 30 40
15
4. In the figure shows a uniform rod of length 30 cm
10
having a mass of 3.0 kg. The strings shown in the
5
figure are pulled by constant forces of 20 N and
0 2 3 4 5 6 8 t(s) 32 N. The force exerted by the 20 cm part of the rod
(a) 0.25 N along the motion, zero and 0.25 N on the 10 cm part is (All the surfaces are smooth and
the strings and the pulleys are light)
opposite to the motion.
10 cm 20 cm
(b) 0.25 N opposite to the motion, zero and 0.25 N
along the motion.
(c) 0.25 N along the motion, zero and 0.25 N along 20 N 32 N
the motion.
(a) 12 N (b) 24 N
(d) 0.25 N opposite to the motion, zero and 0.25 N
opposite to the motion. (c) 48 N (d) 32 N

2. The acceleration of the 500 g block in the figure is 5. A monkey of mass 15 kg is climbing on a rope with
one end fixed to the ceiling. If it wishes to go up with
100 g an acceleration of 1 m/s2, the force it should apply to
the rope is
500 g
30° (a) 125 N (b) 135 N
(c) 165 N (d) 175 N
50 g
6. The force of buoyancy exerted by the atmosphere on
a balloon is B in the upward direction and remains
13 8
(a) g downward (b) g downward constant. The force of air resistance on the balloon
8 13 acts opposite to the direction of velocity and is
8 13 proportional to it. The balloon carries a mass M and
(c) g upward (d) g upward
13 8 is found to fall down near the earth’s surface with
3. Consider the situation a constant velocity v. The mass removed from the
balloon so that it may rise with a constant velocity v
shown in the figure. 1.0 kg 1.0 kg is
All the surfaces are B B
3m  
frictionless and the 5m
4m fixed
(a) 2  M −  (b) 2  M + 
 g  g
string and the pulley
are light. The magnitude of the acceleration of the  B  B
(c) 3  M +  (d) 3  M − 
two blocks is  g  g
4.16 Physics
7. In the figure shows a man of mass 60 9. A small block B is placed on another block A of
kg standing on a light weighing mass 5 kg and length 20 cm. Initially the block B is
machine kept in a box of mass 30 kg. near the right end of block A in the figure. A constant
The box is hanging from a pulley fixed horizontal force of 10 N is applied to the block A.
to the ceiling through a light rope, the All the surfaces are assumed frictionless. The time
other end of which is held by the man elapsed before the block B separates from A is
himself. If the man manages to keep
the box at rest, the weight shown by the machine is B

(a) 10 kg (b) 15 kg 10 N A
(c) 20 kg (c) 25 kg
(a) 0.35 s (b) 0.45 s
8. The monkey B shown in the figure is holding on to
the tail of the monkey A which is climbing up a rope. (c) 0.55 s (d) 0.65 s
The masses of the monkeys A and B are 5 kg and 2 10. The elevator shown in the figure is
kg respectively. If A can tolerate a tension of 30 N in descending with an acceleration
its tail, the force it should apply on the rope in order
of 2 m/s2. The mass of the block
to carry the monkey B with it is (Take g = 10 m/s2) 2
2 m/s
A is 0.5 kg. The force exerted by A
(a) between 50 N and 70 N the block A on the block B is B
(b) between 110 N and 130 N
(a) 4 N (b) 8 N
(c) between 70 N and 105 N
(c) 16 N (d) 20 N
(d) between 140 N and 170 N

Answer Key
1. (a) 2. (b) 3. (a) 4. (b) 5. (c) 6. (a) 7. (b) 8. (c) 9. (b) 10. (a)

6. CONSTRAINT RELATIONS remains the same with time.


Consider the following block-pulley arrangements
(a) Atwood machine
x2
x1
m2

m1
m2
Assume the positions of the blocks m1 m2 as x1, x2
m1 respectively from a fixed reference.
Total length of string = x1 + x2 + portion wrapped
Find the relation between acceleration of block 1 and
around the pulley (pr)
block 2
therefore x1 + x2 + pr = constant
Method 1: By observation: If m1 is displaced
downward by a distance ‘x’ then m2 is displaced the dx1 dx2
differentiating wrt time + +0 =0
same distance ‘x’ upwards in any given time interval, if dt dt
displacements in an time interval are equal and opposite, differentiating wrt time again
the corresponding velocities and accelerations must also v1 + v2 = 0
be equal and opposite a1 + a2 = 0
Method 2: Length of string remains constant: Since This implies that both velocity and acceleration of
ideally the strings are inextensible, the length of string blocks are equal in magnitude and opposite in direction.
Newton’s Laws of Motion 4.17
Method 3: Method of virtual work: The total work 2aP + a2 = 0 ...(ii)
done by tension force on all the objects in the system is Also notice that aQ = a3...(iii)
zero. Using (i), (ii) and (iii), we get
Assume both blocks are displaced by a distance x1 and
a
x2 respectively in the same direction (Say downwards) in a1 + 2a3 + 2 = 0
2
a given time interval. 2a1 + a2 + 4a3 = 0
Method 2: We assume that all masses are displaced
down wards by x1, x2 and x3 respectively.
T
T
T
T m2 x2
T T
x1 m1 2T
x1 P T
m2g m1
x2
m2
m1g 2T 2T
Total work by tension force = – Tx1 – Tx2 = 0 Q
This means x1 + x2 = 0 x3 4T
m3
On differentiating wrt time it gives the required results.
v1 + v2 = 0 Total work done by tension
a1 + a2 = 0 = – 2Tx1 – Tx2 – 4Tx3 = 0
(b) A system with moving pulley we have three masses 2x1 + x2 + 4x3 = 0
m1, m2, m3 and two moving pulley P and Q. The Differentiating twice wrt time
moving pulleys are by default assumed massless 2a1 + a2 + 4a3 = 0
unless stated otherwise. (c) m2 is pulled to the right with constant velocity v2
Find the instantaneousness velocity v1 of m1

P
m1
m2 
m2 v2
Q

m3 v1 m1
Method 1: Instantaneous positions of masses and Method 1: Length of string remains constant
moving pulleys are marked as x1 x2 x3 xP and xQ from fixed
reference. l2 +
x2
2
l l

x1
String 2 x2 m2
x1 xp
x2
P
m1 m1
xQ m2
x1 + l 2 + x22 = constant
Q
x3
differentiation once wrt time
m3 dx1 x2 dx2
String 1 + = 0
dt 2
l +x 2 dt
 dx1 
For string 1: x1 + pr + xQ + pr + xQ – xP = constant 2
 = − v1 
i.e., x1 + 2xQ – xP + 2pr = constant – v1 + v2 cos q = 0  dt 
differentiating twice wrt time
v1 = v2 cos q j
a1 + 2aQ – aP = 0 ...(i) Note: We can’t differentiate again to find acceleration because
For string 2: l + 2xP + pr + x2 = constant dq
we do not know the value of at this instant.
differentiating twice wrt time dt
4.18 Physics
Method 2: Break the velocity v2 into components
Solution:
along the string and perpendicular to the string.

v2 sin 
 2T 2T
m2 u2

v1 m3
v2 cos 
x3
m1 T
T
Velocity (and acceleration) components along string m2
must be equal because the string is inextensible. x2

There fore v1 = v2 cos q m1


x1
(d) Surface constraints (Wedge constraints)
Consider the following examples By virtual work
– Tx1 – Tx2 – 2Tx3 = 0
m1 x1 + x2 + 2x3 = 0
a1 + a2 + 2a3 = 0 …(i)
m2 Draw the free body diagram of the three blocks

T T 2T
Given the objects are rigid and contact is maintained, we
can say that the velocity or acceleration components along m1 m2 m3
the normal to contact surfaces must be equal. a1 a2 a3

m1g m2g m3g

m1g – T = m1 a1 m2g – T = m2a2 m3g – 2T = m3a3



 c os Substitute the expression for a1, a2 and a3 in eqn.
sin  a1 a2 cos 
a1

(i)
a1 a2
 T   T   2T 
 a2 sin   g − m  +  g − m  + 2 g − m  = 0
 1  2  3
Normal components of acceleration of the blocks are equal
24g
a1 cos q = a2 sin q Solving we get T = N
17
a1 = a2 tan q
Substituting back we get the value of accelerations
Illustration 18 24 g 7
a1 = g − =– g m/s 2
A system of three masses m1 = 1 kg, m2 = 2 kg and 17 17
m3 = 3 kg are shown in the figure. The pulleys are 24 g
smooth and massless; the strings are massless and 59
inextensible. a2 = g − 17 = m/s2
2 17
 24 g 
2 
 17  g
a3 = g − = m/s 2
3 17
T2

m3

T1  We have assumed the directions of all the


m2
displacements and accelerations same to simplify
our work. If any acceleration value comes out to
be negative, it would mean that the direction of
m1

(a) Find the tensions in the strings acceleration of that block is opposite to what we
(b) Find the acceleration of each mass. assumed.
Newton’s Laws of Motion 4.19
INTEXT EXERCISE: 4
1. /////////////////////// 5. The acceleration of the block of mass M in the
situation shown in the figure is (All the surface are
frictionless and the pulleys and the string are light)

m2  a2
M
30°
a1  m1

Findout relation between a1 and a2 2M

(a) 2a1 + a2 = 0 (b) a1 + a2 = 0 (a) g/3 down the plane (b) g/3 up the plane
(c) g/2 down the plane (d) g/2 up the plane
(c) a1 + 2a2 = 0 (d) none of these
2. If M1 = 0.3 kg, M2 = 0.2 kg. Then the acceleration of 6. In the figure acceleration of A is
block M2 is 1 m/s2 upwards, acceleration of B
is 7 m/s2 upwards and acceleration
Q
M2 F = 0.4 N of C is 2 m/s2 upwards. Then
M1
acceleration of D will be P

7 4 (a) 7 m/s2 downwards C


(a) m/s 2 (b) m/s 2
2 7 (b) 2 m/s2 downwards D
A B
2 7 (c) 10 m/s2 downwards
(c) m/s 2 (d) m/s 2
7 4 (d) 8 m/s2 downwards
3. The relation between a1 and a2 is (where a1 and a2
are acceleration of block 1 and 2 respectively) 7. The acceleration of block of mass M is

m0
M

5(5M − m0 ) g  5 m0 − M 
2 (a) aM = (b) aM = 5 g
25M + m0  25 m0 − M 
1
 5 m0 + M 
(a) a1 + 7a2 = 0 (b) a1 – 7a2= 0 (c) aM = 5  g
(c) a2 – 7a1 = 0 (d) a2 + 7a1 = 0  25 m0 + M 
 5 m0 + M 
4. In the arrangement shown in figure neglect the (d) aM =  g
masses of the pulley and string and also friction. The  25 m0 + M 
accelerations of blocks A and B are
8. The relation between accelerations of block A, B, C
is

C
A

m1 A B

B m2 C

(a) g, g/2 (b) g/2, g (a) 2aA – aB – 2aC = 0 (b) 2aA + aB + 2aC = 0
(c) 3g/2, 3g/4 (d) g, g (c) 2aA + aB – 2aC = 0 (d) 2aA – aB + 2aC = 0
4.20 Physics
9. The figure shows a rod of length l resting on a wall 10. The velocity of A is
and floor. Its lower end A is pulled towards left with A
a constant velocity v. The velocity of the other end  smooth
B downward when the rod makes as angle q with the
horizontal is
B

l m u
B

A (a) u sin q

u (b) u sec q
(a) VB = u sin θ (b) VB = u cos θ (c) u tan q
(c) VB = u cot θ (d) VB = u tan θ (d) u cot q

Answer Key
1. (c) 2. (c) 3. (d) 4. (d) 5. (b) 6. (c) 7. (a) 8. (b) 9. (c) 10. (b)

7. NON-INERTIAL FRAMES Therefore T = mg


Any reference frame that is accelerating with respect to We know that the magnitude of tension must not
earth or any other inertial frame is celled a non inertial depend on frame of reference and therefore the calculation
frame of reference. of observer 2 is not right.
Lets take an example, observer 1 is at rest on earth. His To correct the calculation, observer 2 must include an
frame of reference is inertial extra force in the F. B. D of mass m. This extra force in not
observer 1 due to interaction with any other object. This is included
as a mathematical correction, so that observer 2 may get
the correct value of tension.
earth Such a force is known as pseudo or fictitious force.
observer 2 is inside an elevator moving up with an Corrected calculation of observer 2:
acceleration ‘a’. According to observer 2 the mass is at rest
hence T = mg + ma
m(g + a)
=
a
m T

observer 2
m
His frame is non-inertial both observers apply newtons ma (Possible force)

laws of motion to calculate the tension in the string


supporting the mass m. mg

According to observes 1 (Inertial frame) Now the value of tension is correct in non-inertial
F.B.D of mass m T frame
According to newtons second law In general, an observer in non-inertial frame must
m a include a pseudo force in the F.B.D. in order to get correct
T – mg = ma results. Without the inclusion of pseudo force, Newton’s
mg
⇒ T = m(g + a) laws would give incorrect results.
T
According to observer 2 (Non-inertial frame) Pseudo Force
F. B. D of mass m m Magnitude: Magnitude of the pseudo force is the product
of mass of object ‘m’ and the acceleration of the observer
For observer 2 the mass is at rest
mg ‘a’. Magnitude of pseudo force: ‘ma’
Newton’s Laws of Motion 4.21
Direction: The direction of pseudo force is opposite to Illustration 20
the direction of acceleration of the observer.
A block of mass m is kept on the incline surface of
Illustration 19 wedge of angle q. Find
A small ball of mass ‘m’ hangs by a chord from ceiling
of a compartment of a train that is accelerating to
the right as shown. Find the tension in the string and m
angle that the string makes with the vertical. M

 a
m (a) The acceleration of the wedge so that the block
remains at at rest relative to it.
Solution: (b) Minimum acceleration of the wedge so that mass
We will show the problem from both inertial and non ‘m’ falls freely all, surfaces are smooth.
inertial frames. Solution:
According to an observer at rest on the ground (Inertial) We will solve the problem from the frame of
T cos  reference of an observer who is at rest relative to wedge.
So if acceleration of wedge is ‘a’ then acceleration of
 observer is also a.
T
(a) block is at rest relative to wedge
a
T sin  
N N sin
a
m

(Pseudo)
mg 
a ma  sin
x direction: ball accelerates with an acceleration ‘a’ sin m
a
mg g 
T sin q = ma...(i)  m sin
g
m
y direction: ball is in equilibrium
wedge accelerates to left with acceleration a
T cos q = mg...(ii)
along the inclined plane:
From (i) and (ii)
mg sin q = ma cos q
2 2
T = m g + a a = g tan q
g along normal to the plane:
Also tan q =
a N = mg cos q + ma sin q
According to an observer in the train compartment
mg
(Non-inertial frame) N = (use a = g tan q)
cos q
According to the observer in the train, the ball is at rest
(b) block falls freely which implies that normal force
x direction: T sin q = ma becomes zero.
y direction: T cos q = mg
N
T cos q
a

ma
T
 mg
T sin 
ma wedge must acceleration to the right
along the normal to the plane:
mg
From the above equations, we get N + ma sin q = mg cos q

g Putting N = 0
2 2
T = m g + a  and tan q = a = g cot q
a
4.22 Physics
INTEXT EXERCISE: 5
1. A frictionless cart carries two other frictionless 6. A block of mass m is placed on a smooth wedge
carts as shown in figure connected by a string over of inclination q. The whole system is accelerated
a pulley. The horizontal force F that must be applied horizontally so that the block does not slip on the
so that m1 and m2 do not move relative to m3 is wedge. The force exerted by the wedge on the block
has a magnitude
m2 g m1
(a) (m1 + m2 + m3 ) (a) mg (b) mg/cos q
m1 F m3 m2 (c) mg cos q (d) mg tan q
m g
(b) (m1 + m2 ) 2
m1 7. A particle stays at rest as seen in a frame. We can
conclude that (Force means only real forces)
m1 g
(c) (m1 + m2 ) (a) the frame is inertial
m2
(b) resultant force on the particle is zero
m1 g
(d) (m1 + m3 ) (c) the frame may be inertial but there is a non zero
m2 resultant force.
2. An object of mass 2 kg moving with constant (d) the frame may be non inertial but there is a
acceleration 2iˆ m/s2 is seen in a frame moving with nonzero resultant force.

constant velocity 10 ˆj m/s. What will be the value of


8. A particle is observed from two frames S1 and
S2. The frame S2 moves with respect to S1 with an
‘pseudo force’ acting on object in this frame. acceleration a. Let F1 and F2 be the pseudo forces on
(a) 20 iˆ N (b) – 20 iˆ N the particle when seen from S1 and S2 respectively.
Which of the following are not possible?
(c) Zero (d) 20 ˆj N
(a) F1 = 0, F2 ≠ 0 (b) F1 ≠ 0, F2 = 0
3. The acceleration of m w.r.t wedge is (c) F1 ≠ 0, F2 ≠ 0 (d) F1 = 0, F2 = 0

Smooth a
9. The mass M of the hanging block in the figure which
m
will prevent the smaller block from slipping over the
 triangular block is (All the surface are frictionless
and the strings and the pulleys are light)
(a) (g + a) sin q (b) (g – a) sin q m
(c) (g + a) cos q (d) (g – a) cos q
M
4. A block is kept on the floor of an elevator at rest. 
The elevator starts descending with an acceleration
of 12 m/s2. The displacement of the block during the
first 0.2 s after the start is [Take g = 10 m/s2] M
(a) 10 cm (b) 20 cm
M′ + m M′ + m
(c) 30 cm (d) 40 cm (a) (b)
tan q − 1 tan q + 1
5. A smooth wedge A is fitted in a chamber hanging
M′ + m M′ + m
from a fixed ceiling near the earth’s surface. A block (c) (d)
B placed at the top of the wedge takes a time T to cot q − 1 cot q + 1
slide down the length of the wedge. If the block 10. The value of a0 so that blocks remains at rest w.r.t
is placed at the top of the wedge and the cable
wedge is (all surfaces are smooth and pulley and
supporting the chamber is broken at the same instant
the block will string is ideal)
(a) take a time longer than T to slide down the m2

wedge
(b) take a time shorter than T to slide down the 
m1
wedge a0

(c) remain at the top of the wedge



(d) jump of the wedge.
Newton’s Laws of Motion 4.23
 m sin α + m2 sin β   m sin α + m2 sin β 
(a) g  1  (c) g  1 
 m2 cos β + m1 cos α   m2 cos β − m1 cos α 
 m sin α − m2 sin β   m sin α − m2 sin β 
(b) g  1  (d) g  1 
 m2 cos β − m1 cos α   m2 cos β + m1 cos α 
Answer Key
1. (a) 2. (c) 3. (a) 4. (b) 5. (c) 6. (b) 7. (d) 8. (d) 9. (c) 10. (a)
4.24 Physics

Unsolved Exercises
EXERCISE - 1
Law’s of Motion 7. A body of mass m1 exerts a force on another body
1. An object will continue moving uniformly until of mass m2. If the magnitude of acceleration of m2
(a) The resultant force acting on it begins to is a2, then the magnitude of the acceleration of m1 is
decrease (considering only two bodies in space)
(b) The resultant force on it is zero ma
(a) Zero (b) 2 2
(c) The resultant force is at right angle to its rotation m1
(d) The resultant force on it is increased continuously m1a2
(c) (d) a2
2. A rope of length L is pulled by a constant force F. m2
What is the tension in the rope at a distance x from
8. A block is placed on a rough horizontal plane. A time
the end where the force is applied
dependent horizontal force F = kt acts on the block.
FL F ( L − x)
(a) (b) Here k is a positive constant. Acceleration‑time
x L
graph of the block is
FL Fx
(c) (d) a a
L−x L−x
(a) (b)
3. Newton’s first law of motion describes the following
(a) Energy (b) Work t t

(c) Inertia (d) Moment of inertia a a


4. A parachutist of weight ‘w’ strikes the ground with
his legs fixed and comes to rest with an upward (c) (d)
acceleration of magnitude 3 g. Force exerted on him t t
by ground during landing is
(a) w (b) 2w 9. Three forces F1, F2 and F3 act on an object
(c) 3w (d) 4w simultaneously. These force vectors are shown in
5. A body is imparted motion from rest to move in a the following free-body diagram. In which direction
straight line. If it is then obstructed by an opposite does the object accelerate?
force, then
(a) The body will not change direction of motion. F1
(b) The body is sure to slow down F2
(c) The body will necessarily continue to move in
the same direction at the same speed
(d) None of these
F3
6. Which figure represents the correct F.B.D. of rod of
mass m as shown in figure :
a
(a) a (b)
smooth surface
T
T
a
(c) (d) a
(a) (b) R
R

10. A body of mass 2 kg is moving along positive


mg mg
X-direction with a velocity of 5 ms–1. Now a force of
T
10 2 N is applied at an angle 45° with X-axis. Its
R velocity after 3s is,
(c) (d) None of these
(a) 20 ms–1 (b) 15 ms–1
mg (c) 25 ms–1 (d) 5 ms–1
Newton’s Laws of Motion 4.25
Particles in Equilibrium 15. A block of weight W is suspended by a string of fixed
11. A mass M is suspended by a ///////////////////////////// length. The ends of the string are held at various
rope from a rigid support at A positions as shown in the figures below. In which case,

A as shown in figure. Another B
F
if any, is the magnitude of the tension along the string
rope is tied at the end B, and largest ?
it is pulled horizontally with a
force F. If the rope AB makes M

an angle θ with the vertical in equilibrium, then the


tension in the string AB is: (a) (b)
(a) F sin θ (b) F/sin θ
(c) F cos θ (d) F/cos θ
12. Two persons are holding a light rope tightly at
its ends so that it is horizontal. A 15 kg weight is
attached to the rope at the mid point which now no
longer remains horizontal. The minimum tension (c) (d)
required to completely straighten the rope is:
15
(a) 15 kg (b) kg
2 Particle Dynamics
(c) 5 kg (d) Ιnfinitely large 16. A person is standing in an elevator. In which situation
(or not possible) he finds his weight less than actual when
13. Ιn the given figure, what is the reading of the spring (a) The elevator moves upward with constant
balance ? acceleration
(b) The elevator moves downward with constant
acceleration.
(c) The elevator moves upward with uniform
velocity
1 kg 1 kg
(d) The elevator moves downward with uniform
(a) 10 N (b) 20 N velocity
(c) 5 N (d) zero 17. A block A of mass 7 kg is
14. A sphere of mass m is kept in equilibrium with the placed on a frictionless A

help of several springs as shown in the figure. table. A thread tied to it


Measurement shows that one of the springs applies a passes over a frictionless B

force F on the sphere. With what acceleration the pulley and carries a body
sphere will move immediately after this particular B of mass 3 kg at the
spring is cut? other end. The acceleration of the system is (given g
= 10 ms–2) –2
(a) 100 ms (b) 3 ms–2
(c) 10 ms–2 (d) 30 ms–2
m 18. Three blocks of masses 2 kg, 3 kg and 5 kg are
connected to each other with light string and are
then placed on a frictionless surface as shown in the
figure. The system is pulled by a force F = 10 N,
then tension T1 =

2 kg 3 kg 5 kg
10 N T1 T2

(a) zero (b) F m
 (a) 1 N (b) 5 N
(c) - F m (d) insufficient information (c) 8 N (d) 10 N
4.26 Physics
19. A closed compartment containing gas is moving If the same force is applied to (m1 + m2), then the
with some acceleration in horizontal direction. acceleration will be:
Neglect effect of gravity. Then the pressure in the (a) 10 ms–2 (b) 2 ms–2
compartment is (c) 2.4 ms–2 (d) 5.4 ms–2
(a) Same everywhere (b) Lower in front side 
25. A force F = 6 iˆ - 8 ˆj + 10 kˆ newton produces
(c) Lower in rear side (d) Lower in upper side
acceleration 1 m/s2 in a body. The mass of the body
20. The value of angle θ such that is (in kg):
the acceleration of A is g/6 A
(a) 6 iˆ - 8 ˆj + 10 kˆ kg (b) 10 2 kg
B
downward along the incline 2m
fixed
m
plane. (All surfaces are smooth) 30° 
(c) 100 kg (d) 10 kg
(a) θ = 30° (b) θ = 60° 26. A body is moving with a speed of 1 m/s and a
(c) θ = 45° (d) θ = 53° constant force F is needed to stop it in a distance x.
21. A force of magnitude F1 acts on a particle so as to If the speed of the body is 3 m/s the force needed to
accelerate it from rest to a velocity v. The force F1 stop it in the same distance x will be
is then replaced by another force of magnitude F2 (a) 1.5 F (b) 3 F
which decelerates it to rest. (c) 6 F (d) 9 F
(a) F1 must be the equal to F2 27. Two blocks, each having mass M, rest on frictionless
(b) F1 may be equal to F2 surfaces as shown in the figure. If the pulleys are
(c) F1 must be unequal to F2 light and frictionless, and M on the incline is allowed
(d) None of these
to move down, then the tension in the string will be:

22. In an imaginary atmosphere, the air exerts a small


force F on any particle in the direction of the particle’s M

motion. A particle of mass m projected upward takes a  fixed M


time t1 in reaching the maximum height and t2 in the /////////////////////////////////////////

return journey to the original point. Then 2 3


(a) Mg sin θ (b) Mg sin θ
(a) t1 < t2 3 2
(b) t1 > t2 Mg sin θ
(c) (d) 2 Mg sin θ
(c) t1 = t2 2
(d) the relation between t1 and t2 depends on the mass 28. A massless spring balance
of the particle. is attached to 2 kg trolley
2 kg

23. Ιn the system shown in the figure, the acceleration of and is used to pull the • •
the 1 kg mass and the tension in the string connecting trolley along a flat surface
between A and B is: as shown in the fig. The reading on the spring
/////////////////// balance remains at 10 kg during the motion. The
acceleration of the trolley is (Use g = 9.8 ms–2)
(a) 4.9 ms–2 (b) 9.8 ms–2
(c) 49 ms–2 (d) 98 ms–2
3 kg C
B 3 kg 29. The ratio of the weight of a man in a stationary
lift & when it is moving downward with uniform
acceleration ‘a’ is 3 : 2. The value of ‘a’ is:
A 1 kg
(g = acceleration due to gravity)
(a) (3/2) g (b) g
g 8g g g
(a) downwards, (b) upwards, (c) (2/3) g (d) g/3
4 7 4 7
30. A block of mass M is placed on a fixed smooth
g 6 g inclined plane of inclination q with the horizontal.
(c) downwards, g (d) upwards, g
7 7 2 The force exerted by the plane on the block has a
24. A force produces an acceleration of 4 ms–2 in a magnitude
body of mass m1 and the same force produces an
(a) Mg (b) Mg/cos q
acceleration of 6 ms–2 in another body of mass m2.
(c) Mg cos q (d) Mg tan q
Newton’s Laws of Motion 4.27
31. A monkey is sitting on the pan of a spring balance 3g
which is placed on an elevator. The maximum (a) g (b)
4
reading of the spring balance will be when:
(a) The elevator is stationary g g
(c) (d)
(b) The string of the elevator breaks and it drops 4 2
freely towards the earth 36. A trolley of mass 5 kg on a A B
(c) The elevator is accelerated downwards horizontal smooth surface
5 kg

(d) The elevator is accelerated upwards. is pulled by a load of mass


32. The adjoining figure shows a force of 40 N pulling 2 kg by means of uniform
C

a body of mass 5 kg in a direction 30° above the 2 kg


rope ABC of length 2 m
horizontal. The body is in rest on a smooth horizontal
surface. Assuming acceleration of free-fall is and mass 1 kg. As the load
10 m/s2. Which of the following statements I and II falls from BC = 0 to BC = 2 m. its acceleration in m/
is/are correct? s2 changes:
40 N 20 20 20 30
(a) to (b) to
30° 6 5 8 8
5 kg
20 30
(c) to (d) None of the above
I. The weight of the 5 kg mass acts vertically 5 6
downwards 37. A pulley is attached to the ceiling of a lift moving
II. The net vertical force acting on the body is 30 N. upwards. Two particles are attached to the two ends
(a) Only I (b) Only II of a string passing over the pulley. The masses of the
(c) Both I and II (d) None of them particles are in the ratio 2 : 1. If the acceleration of the
particles is g/2 w.r.t. elevator then the acceleration of
33. An ideal string is passing over a smooth
the lift will be
pulley as shown. Two blocks m1 and m2
are connected at the ends of the string. If g
(a) g (b)
m1 = 1 kg and tension in the string is 10 2
N, mass m2 is equal to (g = 10 m/s2) g g
m2
(c) (d)
(a) 1 kg (b) 1.5 kg m1 3 4
(c) 2 kg (d) 0.5 kg 38. Two blocks A and B of masses m & 2 m respectively
34. Figures I, II, III and IV depicts variation of force are held at rest such that the spring is in natural
with time length. What is the acceleration of
F(N)
F(N) both the blocks just after release?
0·3 (a) g ↓, g ↓
0·25
I. II. g g
(b) ↓, ↑
1
1 2
3 3
t(ms) t(ms) (c) 0, 0 m A B 2m
F(N) F(N)
(d) g ↓ , 0
1 1
III. IV. 39. In the arrangement shown in figure, pulley is smooth
and massless and all the strings are light. Let F1 be
t(ms) 1 t(ms) 1 the force exerted on the pulley in case (i) and F2 the
force in case (ii) Then
In which situation change in velocity will be
maximum
(a) I & II (b) III & I
(c) III & IV (d) Only IV
35. A monkey is descending from the branch of a tree 4m
2m
4m
m
with constant acceleration. If the breaking strength
m
of branch is 75% of the weight of the monkey, the
minimum acceleration with which the monkey can (a) F1 > F2 (b) F1 < F2
slide down without breaking the branch is (c) F1 = F2 (d) F1 = 2F2
4.28 Physics
40. In the figure, the blocks A, B and C of mass m, each g g
have accelerations a1, a2 and a3 respectively. F1 and (c) (d)
4 2
F2 are external forces of magnitudes 2 mg and mg
45. Two blocks of masses M1 and M2 are connected
respectively.
to each other through a light spring as shown in
figure. If we push mass M1 with force F and cause
acceleration a1 in right direction. What will be the
magnitude of acceleration in M2?
F
m m m M1 M2
A B C ///////////////////////////////////////////////////////////////
m
F1 = 2 mg
2m
F2 = mg
(a) F/M2 (b) F/(M1 + M2)
(c) a1 (d) (F – M1a1)/M2
(a) a1 = a2 = a3 (b) a1 > a3 > a2
(c) a1 = a2, a2 > a3 (d) a1 > a2, a2 = a3 46. Two masses of 10 kg and 20 kg respectively are
connected by a massless spring as shown in figure.
41. A constant force F is applied in horizontal direction A force of 200 N acts on the 20 kg mass at the instant
as shown. Contact force between M and m is N and when the 10 kg mass has an acceleration of 12 ms–2
between m and M′ is N′ then towards right, the acceleration of the 20 kg mass is:
M' > M 10 kg 20 kg
200 N
F M m M'
///////////////////////////////
smooth
(a) 2 ms–2 (b) 4 ms–2
(a) N = N′ (b) N > N′
(c) 10 ms–2 (d) 20 ms–2
(c) N′ > N (d) cannot be determined 47. In the arrangement shown m1
42. A body of mass 8 kg is hanging from another body in the figure all surfaces
of mass 12 kg. The combination is being pulled by a are frictionless, pulley m2 F
string with an acceleration of 2.2 m s–2. The tension and string are light. The
T1 and T2 will be respectively: (use g = 9.8 m/s2) masses of the block are m1 //////////////////////////////////////////////////
= 20 kg and m2 = 30  kg.
T1 The accelerations of masses m1 and m2 will be if
12 kg F = 180 N is applied according to figure.
=
(a) m/s 2 , am2 0
am1 9=
T2 a
=
(b) m/s 2 , am2 9 m/s 2 z
am1 9=
8 kg
am2 9 m/s 2
am1 0,=
(c) =
(a) 200 N, 80 N (b) 220 N, 90 N
(c) 240 N, 96 N (d) 260 N, 96 N (d) None of these
43. A particle of small mass m is joined to a very heavy 48. Three masses of 1 kg , 6 kg and 3 kg are connected
body by a light string passing over a light pulley. to each other with threads and are placed on table as
Both bodies are free to move. The total downward shown in figure. What is the acceleration with which
force on the pulley due to string is nearly the system is moving ? Take g = 10 m s–2.
(a) mg (b) 2mg T1 T2
(c) 4mg (d) can not be determined 6 kg
44. A fireman wants to slide down a rope. The rope can
3
bear a tension of th of the weight of the man. With T1 T2
4
what minimum acceleration should the fireman slide 1 kg 3 kg
down:
g g (a) Zero (b) 1 ms–2
(a) (b)
3 6
(c) 2 ms (d) 3 ms–2
–2
Newton’s Laws of Motion 4.29
  53. In the arrangement shown in fig. the ends P and Q
49. Two forces F1 and F2 act at the ends of a massless
string opposite to each other. Then of an unstretchable string move downwards with
uniform speed U. Pulleys A and B are fixed. Mass M
F1 F2
moves upwards with a speed.
(a) The tension in the string is F1 – F2
(b) Acceleration of the string is zero A B

(c) Tension in the string is F1(= F2)


 Q
(d) Acceleration of the string will be incredibly P 

large
50. Two light strings connect three particles of masses
M
m1, m2 and m3. If m3 moves with an acceleration a as
shown in the figure, the ratio of tension in the strings (a) 2U cos θ (b) U cos θ
1 and 2 is (c) 2U/cos θ (d) U/cos θ
1 2 54. A system is shown in the figure. A man standing on
a
the block is pulling the rope. Velocity of the point
m1 m2 m3
of string in contact with the hand of the man is
(a) m1 : (m1 + m2) (b) m1 : m2 2 m/s downwards. The velocity of the block will be:
(c) m1 – m2 : m1 + m2 (d) m2 : m1 [assume that the block does not rotate]
Constraint Equations
51. A block is dragged on smooth plane with the help of
a rope which moves with velocity v. The horizontal
velocity of the block is:
V

//////////////////
////////////////////////////

(a) 3 m/s (b) 2 m/s


(c) 1/2 m/s (d) 1 m/s

55. Two beads A and B move C
m
along a semicircular wire B
frame as shown in figure. The
v
(a) v (b) beads are connected by an O (centre)
sin θ inelastic string which always
v remains tight. At an instant the A u
(c) v sin θ (d) speed of A is u, DBAC = 45°
cos θ D
and DBOC = 75°, where O is
52. Two masses are connected by a string which passes
the centre of the semicircular arc. The speed of bead
over a pulley accelerating upward at a rate A as B at that instant is:
shown. If a1 and a2 be the acceleration of bodies 2u (b) u
(a)
1 and 2 respectively then:
u 2
A (c) (d) u
2 2 3
R 56. At t = 0, force F = ct is applied to a small body
of mass m resting on a smooth horizontal plane
(c is a constant). The force is at an angle q with the
horizontal .The velocity of the body at the moment of
a1 its breaking off the plane and the distance travelled
a2
1 by the body up to this moment are
2 F
(a) A = a1 – a2 (b) A = a1 + a2

a − a2 a + a2 m
(c) A = 1 (d) A = 1 .
2 2
ct sin  = mg
4.30 Physics
 mg 2 cos θ   mg 3 cos θ  g g
(a)   m/s ,  2 3  m (a) downwards (b) upwards
2  4 4
 2c sin θ   6c sin θ 
g g
 mg 2 cos θ   m 2 g 3 cos θ  (c) downwards (d) upwards
(b)   m/s,  2 3  m 2 2
2 
 2c sin θ   6c sin θ 
Non Intertial Frame of Reference
 mg cos θ   m 2 g 3 sin θ 
(c)   m/s,  2  m 58. Figure shows a wedge of mass 2 kg resting on a
2 3
 2c sin θ   6c cos θ  frictionless floor. A block of mass 1 kg is kept on
 mg 2 cos θ   m 2 g 3 sin θ  the wedge and the wedge is given an acceleration of
(d)  2 
 m/s,  2 3  m 5 m/sec2 towards right. Then:
 2c sin θ   6c cos θ 
57. In the system shown in the adjoining figure, the
acceleration of the 1 kg mass is: 1 kg
2 kg

37°

(a) block will remain stationary w.r.t. wedge


1 kg (b) the block will have an acceleration of 1 m/sec2
w.r.t. the wedge
(c) normal reaction on the block is 11 N
4 kg
(d) net force acting on the wedge is 2 N

EXERCISE - 2

Laws of Motion, Particles in Equilibrium 3. Three blocks A, B and C are suspended as shown
1. A flexible chain of weight W hangs between two in the figure. Mass of each block A and C is m. If
fixed points A and B at the same level. The inclination system is in equilibrium and mass of B is M , then:
///////////////////////////////////
of the chain with the horizontal at the two points of
support is θ. What is the tension of the chain at the
endpoint.
W W A B C
(a) cosec θ (b) sec θ
2 2 (a) M = 2m (b) M < 2m
W (c) M > 2m (d) M = m
(c) W cos θ (d) sin θ
3 4. In the arrangement shown in figure, pulleys are
2. Two masses m and M are attached with strings as massless and frictionless and threads are light and
shown. For the system to be in equilibrium we have inextensible. Block of mass m1 will remain at rest if:
///////////////

/////////////////////

A
M 
///////////

45° 45° B

m
m1 m2 m3
2M 2m 1 1 1 4 1 1
(a) tan θ = 1 + (b) tan θ = 1 + (a) = + (b) = +
m M m1 m2 m3 m1 m2 m3
M m 1 2 3
(c) tan θ = 1 + (d) tan θ = 1 + (c) m1 = m2 + m3 (d) = +
2m 2M m3 m2 m1
Newton’s Laws of Motion 4.31
5. A sphere of mass m is held between two smooth 9. Which graph shows best the velocity-time graph
inclined walls. For sin 37° = 3/5. The normal for an object launched vertically into the air when
reaction of the wall(2) is equal to: air resistance is given by | D | = bv? The dashed
2 line shows the velocity graph if there were no air
1 resistance.
V

37° (a) V t (b) t


37°
(a) mg (b) mg sin 74°
(c) mg cos 74° (d) none of these V V

6. A uniform rope of length L and mass M is placed on a (c) t (d) t


smooth fixed wedge as shown. Both ends of rope are
at same horizontal level. The rope is initially released
from rest, then the magnitude of initial acceleration of 10. A person standing on the floor of an elevator drops
rope is a coin. The coin reaches the floor of the elevator in
time t1 when elevator is stationary and in time t2 if it
is moving uniformly. Then
(a) t1 = t2
(b) t1 > t2
  (c) t1 > t2
(d) t1 < t2 or t1 > t2 depending upon that elevator is
moving upwards or downwards
(a) Zero (b) M(cos α – cos β)g
(c) M(tan α – tan β)g (d) None of these 11. A heavy block kept on a frictionless surface and
being pulled by two ropes of equal mass m as
Subtopics 3: Particles Dynamics shown in figure. At t = 0, the force on the left rope is
7. A force F is applied to the initially stationary cart. withdrawn but the force on the right end continues
The variation of force with time is shown in the to act. Let F1 and F2 be the magnitudes of the forces
figure. The speed of cart at t = 5 sec is by the right rope and the left rope on the block
respectively.

5 kg F Parabolic
50 m m
F F
5
t(s)
(a) F1 = F2 = F for t < 0
(b) F1 = F2 = F + mg for t < 0
(a) 10 m/s (b) 16.3 m/s
(c) F1 = F, F2 = F for t > 0
(c) 2 m/s (d) zero
(d) F1 < F, F2 = F for t > 0
8. A body of mass m resting on a smooth horizontal
plane starts moving under a constant force F. 12. A body of mass 32 kg is suspended by a spring
balance from the roof of a vertically operating lift
During its rectilinear motion, the angle q between
and going downward from rest. At the instants the
the direction of force and horizontal plane varies as lift has covered 20 m and 50 m, the spring balance
q = kx, where k is a constant and x is the distance showed 30 kg and 36 kg respectively. The velocity
travelled by the body from the initial position. What of the lift is:
is the velocity of the body? (a) decreasing at 20 m and increasing at 50 m
F sin θ 2 F sin θ (b) increasing at 20 m and decreasing at 50 m
(a) (b) (c) continuously decreasing at a constant rate
mk mk
throughout the journey
F cos θ 2 F cos θ (d) continuously increasing at constant rate
(c) (d)
mk mk throughout the journey
4.32 Physics
13. Five persons A, B, C, D & E are pulling a cart of 16. A block tied between two springs is in //////////////////////////////

mass 100 kg on a smooth surface and cart is moving equilibrium. If upper spring is cut
with acceleration 3 m/s2 in east direction. When then the acceleration of the block just
person ‘A’ stops pulling, it moves with acceleration
after cut is 6 m/s2 downwards. Now,
1 m/s2 in the west direction. When only person ‘B’ m
if instead of upper spring, lower
stops pulling, it moves with acceleration 24 m/s2 in
the north direction. The magnitude of acceleration spring is cut then the magnitude of
of the cart when only A and B pull the cart keeping acceleration of the block just after
their directions same as the old directions, is: the cut will be: (Take g = 10 m/s2) //////////////////////////////

(a) 26 m/s2 (b) 3 71 m/s2 (a) 16 m/s2


(c) 25 m/s2 (d) 30 m/s2 (b) 4 m/s2
14. There is an inclined surface of inclination θ = 30°. A (c) cannot be determined
smooth groove is cut into it forming angle α with AB. (d) none of these
A steel ball is free to slide along the groove. If the ball
17. Two blocks ‘A’ and ‘B’ each of mass ‘m’ are placed
is released from the point O at top end of the groove,
on a smooth horizontal surface. Two horizontal
the speed when it comes to A is: [g = 10 m/s2]
forces F and 2F are applied on the two blocks ‘A’
and ‘B’ respectively as shown in figure. The block A
Fixed
O does not slide on block B. Then the normal reaction
acting between the two blocks is: (All surfaces are
smooth)
4m 
 90º A B
A B
3m F 2F
m m
30°
(a) 40 m/s (b) 20 m/s //////////////////////////////////////////////////////////////////////////////

(c) 10 m/s (d) 15 m/s (a) F (b) F/2


15. Two particles start together from a point O and slide F
(c) (d) 3F
down along straight smooth inclined planes at 30° 3
& 60° to the vertical & in the same vertical plane as 18. System shown in figure is in equilibrium. The
in figure. The relative acceleration of second with magnitude of change in tension in the string just
respect to first will be (in magnitude and direction) before and just after, when one of the spring is cut.
as: Mass of both the blocks is same and equal to m and
spring constant of both springs is k. (Neglect any
effect of rotation)
/////////////////////////////////////////

k k

m
g
(a) in the vertical direction
2
g 3 m
(b) at 45° with vertical
2
g mg mg
(c) inclined at 60° to vertical (a) (b)
3 2 4
(d) g in the vertical direction 3mg 3mg
(c) (d)
4 2
Newton’s Laws of Motion 4.33
19. In the system shown in figure mA = 4 m, mB = 3 m \\\\\\\\\\\\\
\\\\\\\\\\\\\

and mC = 8 m. Friction is absent everywhere. String F F


is light and inextensible. If the system is released \\\\\\\\\\\\\ \\\\\\\\\\\\\

\\\\\\\\\\

\\\\\\\\\\
from rest find the acceleration of block B (a) (b)
B m m
m m
////////////
A
F
C F
\\\\\\\\\\\\\

\\\\\\\\\\
\\\\\\\\\\\\\
(c) (d)

\\\\\\\\\\
///////////////////////////////////////////
m
g g m
(a) (leftward) (b) (leftward) m m
8 2
24. A light string fixed at one \\\\\\\\\\
g g end to a clamp on ground
(c) (rightward) (d) (rightward)
6 4 passes over a fixed
20. In the arrangement shown pulley and hangs at the a
in figure m1 = 1 kg, M
1 other side. It makes an
m2 = 2 kg. Pulleys are angle of 30° with the 30°
massless and strings are 2 ground. A monkey of \\\\\\\\\\\\\\\\\\\\\\\\\\\\\\\\\
light. For what value of M mass 5 kg climbs up the rope. The clamp can tolerate
the mass m1 moves with m2
m1 a vertical force of 40 N only. The maximum
constant velocity (Neglect acceleration in upward direction with which the
friction) monkey can climb safely is (neglect friction and
(a) 6 kg (b) 4 kg take g = 10 m/s2):
(c) 8 kg (d) 10 kg (a) 2 m/s2 (b) 4 m/s2
21. A trolley is being pulled (c) 6 m/s2 (d) 8 m/s2
up an incline plane by a 25. In figure shown, both blocks are released from rest.
man sitting on it (as The time to cross each other is
shown in figure). He //////////////////////

applies a force of 250 N. 15°

If the combined mass of the man and trolley is 100


kg, the acceleration of the trolley will be [sin 15° =
0.26] 2m
(a) 2.4 m/s2 (b) 9.4 m/s2 4 kg
(c) 6.9 m/s2 (d) 4.9 m/s2 4m

22. If the acceleration of the elevator a0 > g, then 1 kg

(a) 2 second (b) 3 second


(c) 1 second (d) 4 second
26. Same spring is attached with 2 kg, 3 kg and 1 kg
a0
blocks in three different cases as shown in figure.
If x1, x2 and x3 be the extensions in the spring
M when both the bodies possess same magnitude of
m acceleration in these three cases then
(a) the acceleration of the masses will be a0
(b) the acceleration of the masses will be (a0 –g)
mM
(c) the tension in the string will be ( g - a0 )
M +m
(d) tension in the string will be zero. 2 kg 2 kg 3 kg 2 kg 1 kg 2 kg

23. A man thinks about 4 arrangements as shown to


(a) x1 = 0, x3 > x2 (b) x2 > x1 > x3
raise two small bricks each having mass m. Which
of the arrangement would take minimum time? (c) x3 > x1 > x2 (d) x1 > x2 > x3
4.34 Physics
27. Block A of mass m is placed over a wedge B of with the roof of an elevator moving with acceleration
same mass m. Assuming all surfaces to be smooth. a upwards. The acceleration of block M relative to
The displacement of block A in 1 s if the system is M
released from rest is block m is 2a. Ratio is equal to:
m
a a
A (a) 1 + (b) 1 -
B g g
2a 2a
(c) 1 + (d) 1 -
 Fixed g g
31. The pulley shown in the diagram is frictionless. A cat
(1 + sin 2 θ) g sin θ of mass 1 kg moves up on the massless string so as
(a) g 2 (b) to just lift a block of mass 2 kg. After some time,
(1 - sin θ) 2
the cat stops moving with respect to the string. The
cos 2 θ sin 2 θ magnitude of the change in the cat’s acceleration is:
(c) g 2 (d) g
1 + sin θ 1 + sin 2 θ
 
28. Two forces F1 and F2 act on the particles m1 and m2
respectively. If the ideal pulley-particle system
V

remains in smooth horizontal plane, the tension in


the string is:
m1
2 kg

g
m2 (a) g (b)
3
F1 + F2 2g 4g
(a) (b) F1 F2 (c) (d)
2 3 3
m2 F1 + m1F2 | m1F2 - m2 F1 | 32. Consider the Atwood machine as shown in the figure.
(c) (d) The larger mass is stopped for a moment 2.0 s after
m1 + m2 m1 + m2
the system is set into motion. Find the time elapsed
29. In the diagram shown in figure, both pulleys and before the string is tight again.
strings are massless. The acceleration of 2 kg block
is
F = 100 N

1 kg 0.3 kg

0.6 kg
(a) 2/3 sec (b) 1/3 sec
2 kg
3 kg (c) 3/2 sec (d) 1/2 sec
(a) 2.5 m/s2 (↑) (b) 5 m/s2 (↑) 33. A painter of mass M stands on a platform of mass m
and pulls himself up by two ropes which hang over
(c) 7.5 m/s2 (↓) (d) 10 m/s2 (↑)
pully as shown Fig. He pulls each rope with force F
30. Two blocks of masses M and m are connected with and moves upward with a uniform acceleration a.
either end of a massless string which pass over a Find a (neglecting the fact that no one could do this
smooth pulley (M > m). The pulley is connected for long time).
Newton’s Laws of Motion 4.35
38. System is shown in the figure. Velocity of sphere A
is 9 m/s. Then speed of sphere B will be:
A rB = 5m
B

9
m
/s
4 F + ( 2M + m ) g 4F + ( M + m ) g
(a) (b) (a) 9 m/s (b) 12 m/s
M + 2m M + 2m
5
(c) 9 × m/s (d) none of these
4F - ( M + m ) g 4F - ( M + m ) g 4
(c) (d)
M +m 2M + m 39. In the figure shown the blocks A and C are pulled
down with constant velocities u. Acceleration of
34. In Fig both the pulleys and the string are massless
block B is:
and all the surfaces are frictionless.
/////////////////////// ///////////////////////
Given m1 = 1 kg, m2 = 2 kg, m3 = 3 kg. b b
 

A C u
m3 B
m2
m1
u2 u2
(a) tan2 θ sec θ (b) tan3 θ

The tension in the string is b b
120 240 u2
(a) N (b) N (c) sec2 θ tan θ (d) zero
7 7 b
130 40. System is shown in the figure and man is pulling the
(c) N (d) None of these rope from both sides with constant speed ‘u’. Then
7
the speed of the block will be (M moves vertical):
35. In previous problem, the acceleration of m1 is u u
40 30
(a) m/s 2 (b) m/s 2
7 7 /////////////////////////////////////////////

20
(c) m/s 2 (d) None of these
7 ///////////////

36. In previous problem, the acceleration of m3 is M

40 30 3u 3u
(a) m/s 2 (b) m/s 2 (a) (b)
7 7 4 2
20 u
(c) m/s 2 (d) None of these (c) (d) none of these
7 4
41. In the figure shown, find out the value of θ at this
Constraint Equations instant [assume string to be tight ]
37. Block B moves to the right with a constant velocity 1 m/s
v0. The velocity of block A relative to B is:
v0

A B  30° 3 m/s
3.25 m/s
A B
///////////////////////////////////////// ////////////////////////////////////////////////////////////////////////////////
/////////////////////////////////////////////////////////////////

v v 3 −1 4
(a) 0 , towards left (b) 0 , towards right (a) tan
−1
(b) tan
2 2 4 3
3v0 3v 3
(c) , towards left (d) 0 , towards right (c) tan
−1
(d) none of these
2 2 8
4.36 Physics
42. A hinged construction consists of three rhombus 46. What is the relation between velocities of points A,
with the ratio of sides (5 : 3 : 2). Vertex A3 moves in B and C in the given figure.
the horizontal direction with velocity V. Velocity of
A2 will be : //////////////

 A
V U
A1 A2 A3 B
Uniform Speed
(a) 2.5 V (b) 1.5 V
(c) (2/3) V (d) 0.8 V (a) VA = U sec q (b) VA = U cot q
(c) VA = U sin q (d) VB = U cos q
43. System is shown in the figure. Assume that cylinder
remains in contact with the two wedges. The velocity 47. What is the relation between speeds of points A and
of cylinder is B in the given figure.
A
Cylinder
(a) VA = VB cot q
(b) VB = VA cos q
u m/s 2u m/s (c) VA = VB tan q  B
30° 30°
///////////////////////////////////////////////////////////// (d) VA = VB sin q
u 13 u 48. Assuming all the surface to be frictionless,
(a) 19 − 4 3 m/s (b) m/s
2 2 acceleration of the block C shown in the figure is :
2
3 u m/s 7 u m/s
2
(c) (d) 3 m/s 4 m/s
A
B
44. In the arrangement shown in figure pulley A and B
are massless and the thread is inextensible. Mass of
pulley C is equal to m. If friction in all the pulleys is a C
negligible, then
(a) 5 m/s2 (b) 7 m/s2
\\\\\\\\\\\\\\\\\\\\\\\\\\

B
(c) 3.5 m/s2 (d) 4 m/s2
49. System is shown in figure and wedge is moving
A towards left with speed 2 m/s. Then velocity of the
block B will be:
/////////////////////////////////////

/////////////////////////////////////
C
2 m/s B
(a) tension in thread is equal to 1/2mg A

(b) acceleration of pulley C is equal to g/2 //////////////////////////////////////////////////////////////////////////

(downward)
(a) 3 m/s (b) 1 m/s
(c) acceleration of pulley A is equal to g/2 (upward)
(c) 2 m/s (d) 0 m/s
(d) acceleration of pulley A is equal to 2g (upward)
50. In the figure shown the velocity of lift is 2 m/s while
45. In the figure shown block B moves down with a
string is winding on the motor shaft with velocity
velocity 10 m/s. The velocity of A in the position
2 m/s and block A is moving downwards with a
shown is
velocity of 2 m/s, then find out the velocity of block

2 m/s
37°
A
B A
2 m/s
B

(a) 12.5 m/s (b) 25 m/s (a) 2 m/s ↑ (b) 2 m/s ↓


(c) 6.25 m/s (d) None of these (c) 4 m/s ↑ (d) 8 m/s ↑
Newton’s Laws of Motion 4.37
51. Find the acceleration of M shown in figure. Pulleys 54. A pendulum of mass m hangs from a support fixed to
are light and frictionless and strings are light and a trolley. The direction of the string when the trolley
inextensible. (M < 5m) rolls up a plane of inclination α with acceleration
a0 is (String and bob remain fixed with respect to
trolley):
a0

M

m

 5m - M 
(a)   g upwards
 25m + M  
 5m - M 
(b)   g downwards (a) q = tan–1 α
 25m + M 
a 
 5m - M  (b) θ = tan–1  0 
(c)   g upwards  g 
 25m - M 
 g 
 5m - M  (c) θ = tan–1  
(d)   g downwards  a0 
 25m - M 
 a + g sin α 
(d) θ = tan–1  0 
Non-inertial Frames  g cos α 
52. A cuboidal car of height 3 m is  55. A particle is observed from two
m

slipping on a smooth inclined


3

frames S1 and S2. The graph


P

plane. A bolt released from the th of relative velocity of S1 with


oo
roof of car from centre of roof A sm
(P) then distance from centre respect to S2 is shown in figure.
O
of roof where bolt hits the floor ) Let F1 and F2 be the pseudo
with respect to car is : forces on the particle when seen from S1 and S2
(a) 5 m (b) 4 m respectively. Which one of the following is not
(c) 3 m (d) None of these possible?
53. A cylinder rests in a supporting carriage as shown. (a) F1 = 0, F2 ≠ 0 (b) F1 ≠ 0, F2 = 0
The side AB of carriage makes an angle 30o with the (c) F1 ≠ 0, F2 ≠ 0 (d) F1 = 0, F2 = 0
horizontal and side BC is vertical. The carriage lies on 56. A bob is hanging over a pulley inside a car through
a fixed horizontal surface and is being pulled towards a string. The second end of the string is in the hand
left with an horizontal acceleration ‘a’. The magnitude of a person standing in the car. The car is moving
of normal reactions exerted by sides AB and BC of with constant acceleration ‘a’ directed horizontally
carriage on the cylinder be NAB and NBC respectively. as shown in figure . Other end of the string is pulled
Neglect friction everywhere. Then as the magnitude of with constant acceleration ‘a’ (relative to car)
acceleration ‘a ‘ of the carriage is increased, pick up vertically. The tension in the string is equal to
the correct statement:
A C

a
30° B

(a) NAB increases and NBC decreases.


2 2
(a) m g + a
2 2
(b) m g + a - ma
(b) Both NAB and NBC increase.
(c) NAB remains constant and NBC increases.
(d) NAB increases and NBC remains constant. 2 2
(c) m g + a + ma (d) m(g + a)
4.38 Physics
57. A trolley is accelerating down g sin  58. A block is placed on an A
an incline of angle θ with  m inclined plane moving a0 = g
acceleration g sin θ. Which of towards right
the following is correct. (α is horizontally with an
30°
the constant angle made by acceleration a0 = g. The
 B C
the string with vertical). length of the plane AC =
(a) α = θ 1 m. Friction is absent everywhere. The time taken
(b) α = 0° by the block to reach from C to A is  (g = 10 m/s2)
(c) Tension in the string, T = mg (a) 1.2 s (b) 0.74 s
(d) Tension in the string, T = mg sec θ (c) 2.56 s (d) 0.42 s

EXERCISE - 3

Single Option Correct


1. What is the relation between speeds of points A and
B in the given figure.
A

V
V   B
A
B (a) VB + 2VA = 1 (b) – VB – VA = 0
(c) VB – 2VA = 0 (d) 4VB – VA = 0
V (1 + sin q) V (1 - sin q)
(a) VB = (b) VB = 4. A lift of total mass M is raised by cables from rest to
2 cos q cos q rest through a height h. The greatest tension which
V (2 + sin q) V (2 - cos q) the cables can safely bear is nMg. The maximum
(c) VB = (d) VB = speed of lift during its journey if the ascent is to
cot q sin q
made in shortest time is:
2. In the figure acceleration of bodies A, B and C are
 n + 1
shown with directions. Values b and c are w.r.t (a) 2 gh   (b) 2ghn
 n 
ground whereas a is acceleration of block A w.r.t
wedge C. Acceleration of block A w.r.t ground is  n   n -1
(c) 2 gh   (d) 2 gh  
b  n + 1  n 
B
5. Two particles of masses m1 and m2 connected by a
light inextensible string, are released from rest as
a shown in the figure. If the contacting surfaces are
A
C
smooth, at the given position
C
m1


(a) ( b + c )2 + a 2
y
m2
(b) c – (a + b) cos q
x
2 2
(c) (b + c) + c - 2(b + c) c cos q
a1 m1 a1x m2
(a) = (b) = sec q
(d) (b + c ) 2
+ c + 2 ( b + c ) c cos q
2 a2 m2 a2 x m1

3. What is the relation between velocities of points A v1


(c) = sec q (d) a1 = a2 cos q
and B in the given figure. v2
Newton’s Laws of Motion 4.39
6. In an elevator going up with V (VA - VB ) cos q1
(a) Vc = - VA
constant velocity V, particle A at cos q2
the end of a smooth incline and B
(VA + VB ) cos q1
particle B from the height at the (b) Vc = - VB
L A cos q2
other end of the incline are C 
(VA + VB ) sin q1
released at the same time, as (c) Vc = + VB
sin q2
shown. Particle B is dropped vertically. At the end C
(VA - VB ) cos q2
of the incline, particles collide. Find the height by (d) Vc = + VB
2 cos q1
which Bs dropped.
(a) L cos q (b) L sin q 10. If the block A and B are moving towards each other
(c) L tan q (d) L cosec q with acceleration a and b as shown in the figure.
Find the net acceleration of block C.
7. In the figure the heavy mass m moves down the
smooth surface of a wedge making an angle a with
A B
the horizontal. The wedge is on a smooth surface. C
a b
The mass of the wedge is M. The direction of motion
of the mass m makes an angle b with the horizontal, (a) aiˆ - 2(a + b) ˆj (b) - a(a + b) ˆj
then ‘tan b’ is
(c) ajˆ - (a + b) ˆj (d) None of these
m
11. The acceleration of the block B in the above figure,
M
assuming the surfaces and the pulleys P1 and P2 are
all smooth.
2m 4m
m M
(a) tan α (b) tan α P1
M m F A
P2
B

 m  M
(c) 1 +  tan α (d) 1 +  tan α F F
 M  m (a) (b)
4m 6m
8. The rod A slides down on block B which slides down F 3F
on a fixed inclined plane making an angle q with (c) (d)
2m 17m
horizontal. The angles are shown. If aA and aB are
a 12. The system starts from
the acceleration of A and B respectively, then B is rest and A attains a
aA A 6 kg F
velocity of 5 m/s after it
A has moved 5 m towards
right. Assuming the B 2 kg
arrangement to be
B frictionless every where and pulley and string to be
90° light, the value of the force F applied on A is :
(a) 50 N (b) 75 N
 (c) 100 N (d) 96 N
(a) sin q (b) cos q 13. Two insects each of mass m moves A
g
(c) tan q (d) cot q with accelerations a1 = a2 =
2
9. What is the relation between velocities of points A, B a2
B and C in the given figure. relative to the light inextensible
string as shown in the figure. The
a1 C
ratio of tensions in the portions AB
VC
and BC of the string is:
VA
1
B VB 
2
(a) 1 : 2 (b) 3 : 1
A C
(c) 4 : 1 (d) 2 : 1
4.40 Physics
14. A smooth ring of mass m can A 18. In the given fig, pulley is frictionless and massless.
slides on a fixed horizontal m  Both the springs are having same force constant
rod. A string tied to the ring B
10  N/m. Initially with the string attached to the
passes over a fixed pulley B grounds, the total system is at rest. Now if the string
and carries a block C of mass C is cut, then immediately after cutting the string,
2m as shown below. As the M
ring starts sliding:
2 g cos q
(a) The acceleration of the ring is
1 + 2cos 2 q
2 g cos 2 q
7 kg 5 kg
(b) The acceleration of the block is
1 + 2 cos 2 q
K = 10 N/m K = 10 N/m
4 mg
(c) The tension in the string is
1 + 2 cos 2 q 5 kg 4 kg
String
(d) If the block descends with velocity v then the
ring slides with velocity v cos q. (a) acceleration of 7 kg block is 5 m/s2
15. In the figure shown, friction is absent. The system (b) acceleration of 5 kg block is zero
moves with constant speed. Then ‘m′’ is equal to (c) acceleration of 5 kg block is 6 m/s2
v = const (d) acceleration of 4 kg block is 10 m/s2
l
 Multiple Options Correct
M
m
m' 19. A wedge of mass M is loaded with a sphere of mass
 m. If a horizontal force F is applied on the wedge, it
remains in equilibrium. Then:
(a) (M + m) sin a (b) M sin a + m
sin α
(c) M + sin a (d) ( M + m) m
sin b F 

16. A slider block of mass 10  kg with a hanging rope M


is moving on an inclined surface at an angle of 30°
with horizontal. A monkey of 10 kg is climbing on
the rope with an acceleration of 1 m/s2. The slider (a) N1 = mg sec q; N1 = reaction force between
block also slides with acceleration of 0.5 m/s2. The wedge and sphere
surface are smooth. How much force should the (b) F = mg tan q
monkey apply to the rope to climb (g = 10 m/s2) (c) N2 = mg tan q; N2 = reaction force between
(a) 107.5 N (b) 115.5 N sphere and vertical wall
(c) 127.5 N (d) 105.5 N (d) N3 = Mg + mg; N3 = reaction force between
ground and wedge
17. In figure all the pulleys and strings are massless and
20. In the adjoining figure if acceleration of M with
all the surfaces are frictionless. Small block of mass
respect to ground is a, then:
m is placed on fixed wedge. (take g = 10 ms–2) The
acceleration of pully p4 is
P1
F = 80 N

P5 P2
P3 a m
P4
P7 M

m = 10 kg
P6
30° 
(a) Acceleration of m with respect to M is a
(a) 2.25m/s2 towards left (b) Acceleration of m with respect to ground is
2
(b) 2.25 m/s towards right 2a sin (a/2)
(c) 9 m/s2 towards left (c) Acceleration of m with respect to ground is a
(d) 9 m/s2 towards right (d) Acceleration of m with respect to ground is a tan a
21. Two blocks 1 and 2 being fitted with the pulleys
move with velocities v1, v2 and accelerations a1 and
a2 as shown in the figure. If the block Q is connected
with the block 2 with an inextensible string that
passes over the pulleys, which of the following is/
//////
Newton’s Laws of Motion 4.41

A
B

are correct regarding the velocities and accelerations


of the point P and the block Q ?
a1 a2
v2 (a) The maximum acceleration that painter can
v1
P have upwards is 5 m/s2.
1 2 (b) To hoist himself up, rope B must withstand
minimum 200 N force.
Q
(c) Rope A will have a tension of 100 N when the
(a) vp = 2v1 – v2 (b) ap = – (2a1 + a2)

/////
painter is at rest.
(c) vQ = – 3v2 + 4v1 (d) aQ = – (3a2 + 4a1) (d) The painter must exert a force of 200 N on the
22. A block of mass m is placed on a wedge. The wedge rope A to go downwards slowly.
is accelerating with same acceleration in the cases
given as (1), (2), (3) and (4) as shown. If the normal 25. Two men of unequal masses hold on to the two
reactions in situation (1), (2), (3) and (4) are N1, N2, sections of a light rope passing over a smooth light
N3 and N4 respectively and acceleration with which pulley. Which of the following are possible?
the block slides on the wedge in situations are b1, b2,
b3 and b4 respectively then :

m m
37° 37°
(1) (2)

(a) The lighter man is stationary while the heavier


m m man slides with some acceleration
37° 37° (b) The heavier man is stationary while the lighter
man climbs with some acceleration
(3) (4)
(c) The two men slide with the same acceleration in
(a) N3 > N1 > N2 > N4 (b) N4 > N3 > N1 > N2
the same direction
(c) b2 > b3 > b4 > b1 (d) b2 > b3 > b1 > b4
(d) The two men move with accelerations of the
23. A pendulum bob is hanging from a pulley P by an same magnitude in opposite directions
inextensible light thread which is connected with the 26. Figure shows two blocks A and B connected to an
cart after passing over the pulley. If the cart moves ideal pulley string system. In this system when
down the inclined plane, then: bodies are released then :
(neglect friction and take g = 10 m/s2)
P M 10 kg
m B


(a) Net force acting on the system (M + m) is
(M + m)g sin q
(b) Tension in the thread is mg cos q
(c) Force acting on the pulley by the thread is
2 mg cos q
a
(d) Force acting on the cart by the thread is mg cos q 40 kg A

24. To paint the side of a building, painter normally (a) Acceleration of block A is 1 m/s2
hoists himself up by pulling on the rope A as in (b) Acceleration of block A is 2 m/s2
figure. The painter and platform together weigh (c) Tension is string connected to block B is 40 N
200 N. The rope B can withstand 300 N. Then (d) Tension in string connected to block B is 80 N
4.42 Physics
27. In the system shown in the figure m1 > m2 . System y
is held at rest by thread BC. Just after the thread BC A
is burnt : 
/////////////////////
B x
C


spring B m2 Horizontal Surface
k

(a) Acceleration of ‘A’ relative to ground is in


m1 A
////////////
negative y-direction
C (b) Acceleration of ‘A’ relative to B is in positive
(a) acceleration of m2 will be upwards x-direction
(b) magnitude of acceleration of both blocks will be (c) The horizontal acceleration of ‘B’ relative to
ground is in negative x-direction.
 m - m2 
equal to  1 g (d) The acceleration of ‘B’ relative to ground
 m1 + m2  directed along the inclined surface of ‘C’ is
(c) acceleration of m1 will be equal to zero greater than g sin θ.
(d) magnitude of acceleration of two blocks will be 31. A block B of mass 0.6 kg slides down the smooth
non-zero and unequal. face PR of a wedge A of mass 1.7 kg which can
28. A light string is wrapped round a cylindrical log of move freely on a smooth horizontal surface. The
wood which is placed on a horizontal surface with inclination of the face PR to the horizontal is 45°.
it’s axis vertical and it is pulled with a constant Then :
force F as shown in the figure. (Friction is absent P
everywhere)
B
F A
Q 45° R

(a) tension T in the string increases with increase (a) the acceleration of A is 3 g/20
in θ
(b) the vertical component of the acceleration of B
(b) tension T in the string decreases with increase
in θ is 23 g/40
(c) tension T > F if θ > π/3 (c) the horizontal component of the acceleration of
(d) tension T > F if θ > π/4 B is 17 g/40
29. According to figure the reading of the spring balance (d) none of these
will be: (all contacts are smooth) [g = 10 m/s2]
2
2 m/s 32. In the Figure, the pulley P moves to the right with
a constant speed u. The downward speed of A is vA,
and the speed of B to the right is vB.
B P
10 kg
u
5 kg
30° Fixed
A vA
(a) 6 kgf (b) 5 kgf
(c) 60 N (d) 60 kgf (a) vB = vA
(b) vB = u + vA
30. In the figure shown all the surface are smooth. All the
blocks A, B and C are movable X-axis is horizontal (c) vB + u = vA
and y-axis vertical as shown. Just after the system is (d) the two blocks have accelerations of the same
released from the position as shown. magnitude
Newton’s Laws of Motion 4.43
33. In the arrangement shown in figure all surfaces are 36. If acceleration of block A is a rightward, then
smooth. Select the correction alternative(s): acceleration of block B will be:
3a 4a
(a) upwards (b) upwards
B 4 3
A
3a 4a
Fixed (c) upwards (d) upwards
 5 5
(a) for any value of q acceleration of A and B are Passage 2 (Questions 37–38)
equal
(b) contact force between the two blocks is zero if
mA/mB = tan q
(c) contact force between the two is zero for any a
value of mA or mB m1

(d) normal reactions exerted by the wedge on the m2

blocks are equal 37. An insect of mass m1, equilibrates a hanging mass
Comprehension Type m2 by moving relative to the string with an upward
acceleration a. The value of m1 is:
Passage 1 (Questions 34 to 36)
m1 ga m2 a
Two smooth blocks are placed at a smooth corner as (a) (b)
g+a g
shown. Both the blocks are having mass m. We apply a
force F on the small block m. Block A presses the block g+a m2 g
(c) m2 (d)
B in the normal direction, due to which pressing force g-a g+a
on vertical wall will increase, and pressing force on the 38. If the insect moves up relative to the string with
horizontal wall decreases, as we increase F. (q = 37° with constant velocity, the acceleration of m2 is
horizontal). As soon as the pressing force on the horizontal |a - g |
(a) g (b) ga
wall by block B becomes zero, it will loose the contact with a+g
the ground. If the value of F is further increased, the block g+a ga
B will accelerate in upward direction and simultaneously (c) .g (d)
| g - a| 2g + a
the block A will move toward right.
y Passage 3 (Questions 39 to 42)
B In the system shown in figure, mA = 4 m, mB = 3 m, and
mC = 8 m. Friction is absent everywhere, and the string is
smooth inextensible and light. If the system is released from rest,
A m then find the following.
F m B
 = 37°
x
A
34. What is minimum value of F, to lift block B from C
ground:
25 5
(a) mg (b) mg
12 4 39. The tension in the string is
3 4 (a) 1.5 mg (b) 5.8 mg
(c) mg (d) mg
4 3 (c) 4.7 mg (d) 3.2 mg
35. If both the blocks are stationary, the force exerted by 40. The acceleration of Block C is
ground on block A is : (a) 1/4 m/s2 (b) 2/7 m/s2
(c) 5/4 m/s2 (d) 1/3 m/s2
3F 3F
(a) mg + (b) mg -
4 4 41. The acceleration of Block B is
4F 4F (a) 20 m/s2 (b) 5 m/s2
(c) mg + (d) mg -
3 3 (c) 15 m/s2 (d) 10 m/s2
4.44 Physics
42. The acceleration of Block A in horizontal direction 47. A cabin is supported by a rope which ///////////////////////
is passes over a frictionless fixed pulley.
5 21 A man weighing 1000 N stands on the
(a) m/s2 (b) m/s2 chair and exerts a force of 450 N on the
4 2
cabin downwards while pulling the
11 27 other end of the rope. The chair weighs
(c) m/s2 (d) m/s2
7 5 250 N. Find the acceleration of cabin
Passage 4 (Questions 43 to 45) in m/sec2 (g = 10 m/s2).
48. A block of mass m = 6 kg sits on a plank of mass
Figure shows a weighing machine kept in a lift.
M = 10 kg which lies on a smooth surface. The
Lift is moving upwards with acceleration of
coefficient of friction between the block and the
5 m/s2. A block is kept on the weighing machine. Upper
plank is n = 0.5 . At t = 0 , a time dependent force
surface of block is attached with a spring balance. Reading
F = 2t starts to act on the plank. If the block starts to
shown by weighing machine and spring balance is 15 kg
slip on the plank at t = xg (where g is acceleration
and 45 kg respectively.
due to gravity), find the value of x?
Answer the following questions. Assume that the
weighing machine can measure weight by having m
F
negligible deformation due to block, while the spring M

balance requires larger expansion: (take g = 10 m/s2) Smooth

49. A sphere of mass kg is placed on two inclined


Spring
Balance planes of angle 30° and 60° with horizontal, as
? shown. Find the normal reaction at point P (in N)

M P
60° 30°
Weighing
Machine 50. In a smooth hemispherical shell of radius R, a rod of
43. Mass of the object in kg and the normal force acting  3
on the block due to weighing machine are: mass   kg is placed horizontally and is in
 2 
(a) 60 kg, 450 N (b) 40 kg, 150 N
(c) 80 kg, 400 N (d) 10 kg, zero equilibrium. The length of rod is R. Find the normal
reaction at any end of the rod (in N).
44. If lift is stopped and equilibrium is reached. Reading
[Take g = 10 m/s2]
of weighing machine and spring balance will be:
51. A small projectile is to be launched with a fixed speed
(a) 40 kg, zero (b) 10 kg, 20 kg
from left to right. There is a strong wind blowing
(c) 20 kg, 10 kg (d) zero, 40 kg from the right to left that causes an acceleration
45. Find the acceleration of the lift such that the equal to that of gravity to the left. For maximum
weighing machine shows its true weight. range, the projectile is launched at an angle i = r/m
45 85 from the horizontal. What is the value of m?
(a) m/s 2 (b) m/s 2
4 4 52. A block is moving on an inclined plane making
22 60 an angle of 45c with the horizontal surface and
(c) m/s 2 (d) m/s 2 the coefficient of friction is n . The force required
4 4
to push up the inclined plane, is 3 times the force
Integer Type required to prevent it from sliding down. If we
46. a pulley system is arranged as shown. /////////////////////// define N = 10n , then find the value of N.
Man A stands on the plank and moves 53. A groove AB = 1.16 m is cut in
up and down with it. Man B is at rest plane of an inclined plane of  B
53
inclination of 37th. A block is
on the ground. Both men allow rope A
touching every surface of the
to pass through their hands as the A
groove starts slide from point B.
37

plank moves. If the ratio of length of The coefficient of kinetic friction between the block
rope passing through the hands of A B
and all surfaces of groove is 0.1. Find the time (in
and B is x/3. what is the value of x? second) taken by the block to come at point A.
///////////////////////
Newton’s Laws of Motion 4.45
Matching Column Type 55. The system shown below is initially in equilibrium.
Masses of the blocks A, B, C, D and E are respectively
54. Two blocks A and B of masses 2 kg and 3 kg are 3m, 3m, 2m, 2m and 2m. Match the conditions in
placed on horizontal surface. The coefficient of
friction between block A and horizontal surface is 0.5 column I with the effects in column I.
while the block B is smooth. A force F = ^ 2 t h N
is applied on block A as shown in the figure.
F C
Spring 1

45 B D
A B
Spring 2

A E

Match the column I with Column II and choose the
correct option from the codes given below.
Column I Column II
Column I Column II
(a) After spring 2 is cut, tension (p) increases
(a) t = 4s (p) The acceleration of block B in string AB
is zero.
(b) After spring 2 is cut, tension (q) decreases
(b) t = 20s (q) Friction on block A is more in string CD
than 20 N (c) After string between C and (r) remain
(c) t = 40s (r) The acceleration of block A pulley is cut, tension in constant
is 2 m/s2 string AB

(d) (s) Normal contact force (d) After string between C and (s) zero
t = 60s
between blocks A and B is pulley is cut, tension in
non – zero. string CD

1. A particle is moving with a velocity v = K _yi^ + xj^ i , F0


mb
F0b
mb
where K is a constant. The general equation for its (a) (b)
path is v(t) v(t)

[2019] t t

(a) y2 = x2 + constant (b) y = x2 + constant F0


F0
(c) y2 = x + constant (d) xy = constant mb mb
(c) (d)
2. A mass of 10 kg is suspended vertically by a rope v(t) v(t)

from the roof. When a horizontal force is applied on t t

the rope at some point, the rope deviated at an angle


of 45° at the roof point. If the suspended mass is at 4. If a spring of stiffness ‘k’ is cut into two parts ‘A’ and
equilibrium, the magnitude of the force applied is (g ‘B’ of length lA : lB = 2 : 3, then the stiffness of spring
= 10 m s–2) ‘A’ is given by:
[2019]
(a) 140 N (b) 70 N [2011]
(c) 100 N (d) 200 N
3k 2k
3. A particle of mass m is at rest at the origin at time (a) (2)
5 5
t = 0. It is subjected to a force F(t) = F0e–bt in the x
direction. Its speed v(t) is depicted by which of the 5k
(c) k (d)
following curves? [2012] 2
4.46 Physics
5. Two fixed frictionless inclined planes making an (a) g tan a (b) g cot a
angle 30° and 60° with the vertical are shown in the (c) g sec a (d) none of these
figure. Two blocks A and B are placed on the two
9. Two masses m1 = 5 kg and m2 = 4.8 kg
planes. What is the relative vertical acceleration of
tied to a string are hanging over a light
A with respect to B? [2010]
frictionless pulley. What is the
A acceleration of the masses when
system is free to move ?
B m1
(g = 9.8 m/s2 )[2004]
m2
(a) 0.2 m/s2 (b) 9.8 m/s2
60° 30°
(c) 5 m/s2 (d) 4.8 m/s2
(a) 4.9 ms–2 in horizontal direction 10. A spring balance is attached to the ceiling of a lift.
(b) 9.8 ms–2 in vertical direction A man hangs his bag on the spring and the spring
reads 49 N. When the lift is stationary. If the lift
(c) Zero
moves downward with an acceleration of 5 m/s2, the
(d) 4.9 ms–2 in vertical direction reading of the spring balance will be: [2003]
6. A block of mass m is connected to another block of (a) 24 N (b) 74 N
mass M by a string (massless). The blocks are kept (c) 15 N (d) 49 N
on a smooth horizontal plane. Initially the blocks
are at rest. Then a constant force F starts acting on 11. A block of mass M is pulled along a horizontal
the block of mass M to pull it. Find the force on the frictionless surface by a rope of mass m. If a force
block of mass m[2007] P is applied at the free end of the rope, the force
mF ( M + m) F exerted by the rope on the block is:[2003]
(a) (b)
m m Pm Pm
(a) (b)
mF MF M +m M -m
(c) (d)
(m + M ) (m + M ) PM
(c) P (d)
M +m
7. A ball of mass 0.2 kg is thrown vertically upwards
by applying a constant force by hand. If the hand 12. A light spring balance hangs from the hook of the
moves 0.2 m while applying the force and the ball other light spring balance and a block of mass M kg
goes upto 2 m height further, find the magnitude of hangs from the former one. Then the true statement
the force. Consider g = 10 m/s2. [2006] about the scale reading is:[2003]
(a) 20 N (b) 22 N (a) Both the scale read M kg each
(c) 4 N (d) 16 N (b) The scale of the lower one reads M kg and of the
8. A block is kept on a frictionless inclined surface upper one zero
with angle of inclination α. The incline is given an (c) The reading of the two scales can be anything
acceleration a to keep the block stationary. The a is but the sum of the reading will be M kg
equal to [2005]
(d) Both the scales read M/2 kg

Single Option Correct of the new equilibrium position of the bead, where
the bead can stay at rest with respect to the wire,
1. A piece of wire is bent in the shape of a parabola y =
from the y-axis is [2009]
kx2 (y-axis vertical) with a bead of mass m on it. The
a a
bead can slide on the wire without friction. It stays (a) (b)
gk 2 gk
at the lowest point of the parabola when the wire is
at rest. The wire is now accelerated parallel to the 2a a
(c) (d)
x-axis with a constant acceleration a. The distance gk 4 gk
Newton’s Laws of Motion 4.47
2. Two particles of mass m each are tied at the ends of a 3. System shown in figure is in equilibrium
light string of length 2a. The whole system is kept on and at rest. The spring and string are
a frictionless horizontal surface with the string held massless Now the string is cut. The
tight so that each mass is at a distance ‘a’ from the
acceleration of mass 2m and m just after
centre P (as shown in the figure). Now, the mid-point
of the string is pulled vertically upwards with a small the string is cut will be: [2006] 2m

but constant force F. As a result, the particles move (a) g/2 upwards, g downwards
towards each other on the surface. The magnitude m
(b) g upwards, g/2 downwards
of acceleration, when the separation between them
becomes 2x, is [2007] (c) g upwards, 2g downwards
F (d) 2g upwards, g downwards
4. The pulleys and strings shown in the figure are
smooth and of negligible mass for the system to
m P m remain in equilibrium, the angle θ should be [2002]
a a

F a F x


(a) (b)
2m a2 - x2 2m a2 - x2 2m

m m

F x F a2 - x2 (a) 0º (b) 30º


(c) (d)
2m a 2m x (c) 45º (d) 60º
4.48 Physics
Answer Key
Exercise 1

1. (b) 2. (b) 3. (c) 4. (d) 5. (b) 6. (c) 7. (b) 8. (c) 9. (d) 10. (c)

11. (b) 12. (d) 13. (a) 14. (c) 15. (a) 16. (b) 17. (b) 18. (c) 19. (b) 20. (a)

21. (b) 22. (b) 23. (c) 24. (c) 25. (b) 26. (d) 27. (c) 28. (c) 29. (d) 30. (c)

31. (d) 32. (a) 33. (a) 34. (b) 35. (c) 36. (b) 37. (b) 38. (a) 39. (c) 40. (b)

41. (b) 42. (c) 43. (c) 44. (c) 45. (d) 46. (b) 47. (a) 48. (c) 49. (c) 50. (a)

51. (b) 52. (c) 53. (d) 54. (b) 55. (a) 56. (b) 57. (d) 58. (c)

Exercise 2

1. (a) 2. (a) 3. (b) 4. (b) 5. (a) 6. (a) 7. (b) 8. (b) 9. (b) 10. (a)

11. (a) 12. (b) 13. (c) 14. (a) 15. (a) 16. (b) 17. (d) 18. (a) 19. (b) 20. (c)

21. (d) 22. (d) 23. (a) 24. (c) 25. (c) 26. (b) 27. (d) 28. (c) 29. (a) 30. (c)

31. (c) 32. (a) 33. (c) 34. (a) 35. (a) 36. (b) 37. (b) 38. (b) 39. (b) 40. (a)

41. (a) 42. (d) 43. (d) 44. (d) 45. (b) 46. (d) 47. (a) 48. (c) 49. (c) 50. (d)

51. (a) 52. (c) 53. (c) 54. (d) 55. (d) 56. (c) 57. (a) 58. (b)

Exercise 3

1. (b) 2. (c) 3. (c) 4. (d) 5. (b) 6. (d) 7. (c) 8. (a) 9. (b) 10. (a)

11. (d) 12. (b) 13. (b) 14. (b) 15. (a) 16. (a) 17. (b) 18. (b) 19. (a, b, c, d)

20. (a, b) 21. (a, b, c, d) 22. (a, c) 23. (a, b, c, d) 24. (a, b, c) 25. (a, b, d)

26. (b, d) 27. (a, c) 28. (a, c) 29. (a, c) 30. (a, b, c, d) 31. (a, b, c)
32. (b, d)

33. (a, c) 34. (c) 35. (c) 36. (a) 37. (d) 38. (d) 39. (a) 40. (c) 41. (b) 42. (a)

43. (b) 44. (d) 45. (a) 46. (4) 47. (2) 48. (4) 49. (5) 50. (5) 51. (8) 52. (5)

53. (1) 54. a → (p), b → (p), c → (q, r, s), d → (q, s) 55. a → (r), b → (q), c → (s), d → (r)

Archives: Least Attempted Questions (LAQs) (JEE Main)

1. (a) 2. (c) 3. (c) 4. (d) 5. (d) 6. (c) 7. (b) 8. (a) 9. (a) 10. (a)

11. (d) 12. (a)

Archives: Least Attempted Questions (LAQs) (JEE Adv)

1. (b) 2. (b) 3. (a) 4. (c)

You might also like